You are on page 1of 85

Section I

The Bread and Peanut


Butter Stations
(60 Stations)

Notes:
These are the stations that you should have down cold. The majority
of the stations will likely come from these examples.
Questions were culled from the CFPC 99 Topics of Family Medicine

Combined Stations (7 Stations)


65yo F comes in with a decrease in hearing. Take a hx and px.
Conductive vs Sensorineural hearing loss: obstruction, otitis media, TM perforation,
presbycusis, noise, ototoxic drugs (aminoglycosides, furosemide), Meniere's dz
(min-hrs; triad of tinnitus, hearing loss, vertigo), MS, CVA, congenital (TORCH)

1. HPI
Onset, sudden/gradual, both ears
Hearing voice not understanding, men's voice easier than women's
Pain/fullness, discharge, ringing, vertigo
Congestion, headache
Dry eyes/mouth, oral ulcers, joint pain
2. Meds/OTC/Allergies
Aminoglycosides, furosemide
3. PMHx/Surgery/Hospitalization
DM, HTN
Head injuries, stroke
Ear syringed, hearing test, ear infections
4. Social Hx
Smoking/ETOH/IVDU
Occupation, noise exposure, ear protection
5. Family Hx
Hearing loss, Meniere's dz, MS
Physical Examination
Vital signs
1. Inspection
Asymmetry, bruising, mass, tenderness, rash
Nasopharynx masses
2. Ear Exam
Otoscope
Finger rub test, Weber's (lateralization), Rinne's (AC>BC)
3. CN Exam
II, III/IV/VI, V (corneal reflex), VII [cerebellopontine angle ]
Pronator drift, finger-to-nose, repetitive movements
25yo M sprain ankle while running. Do a focused hx and px exam.
Anterior/posterior talofibular, calcaneofibular strain, fibular #, tibial avulsion #,
metarsal/calcaneal stress #, base of 5th metatarsal #, arthritis, tendonitis (posterior
tibial, peroneal), plantar fasciitis, tarsal tunnel syndrome, bony heel spur, Morton's
neuroma

1. HPI
Mechanism of injury, previous injury, wt bearing, pain (PQRSTPP)
Swelling, skin changes, stiffness/locking/grinding
Head injury, vertigo, visual changes

CP/SOB, palpitations
2. Meds/OTC/Allergies
Steroids, pain killers
3. PMHx/Surgery/Hospitalization
DM, HTN
Previous falls
Bleeding d/o, osteoporosis
4. Social Hx
Smoking/ETOH/IVDU, occupation, handedness
Physical Exam
1. Inspection
Examine hands for nodes (Heberden's DIP and Bouchard's) and OA
Exposes up to knee, gait, heel/toe walk,
Swelling, Erythema, Atrophy, Deformity (symmetry of feet, toe
alignment, foot arches, heel alignment [varus/valgus], Achilles
tendon), Skin (color, warmth, scars, lesions, wounds)
2. Palpation
Compare temperature, pulses,
Squeeze MTPs, palpate Achilles tendon
Lateral/medial malleoli, proximal fibula, base of 5th metatarsal,
navicular
Ankle joint, tarsal joint, subtalar (talocalcaneal joint) joint,
3. ROM
Plantar/dorsiflexion, inversion/eversion
Toe flexion, extension, adduction, abduction
Anterior drawer test (ATFL), Thompson test
Examine Knee/Hip
4. Neuro
Tone, Motor (L2 hip flexion; L3 knee extension; L4 inversion of
foot; L5 extension of great toe; S1 eversion of foot)
Sensory (L2 lateral thigh; L3 middle knee; L4 middle ankle; L5
1st web space; S1 lateral foot) [light touch, pin-prick, vibration]
Reflexes (L4 patellar; S1 achilles, Babinski, clonus)
PEP Questions

What are the 3 views of the ankle? AP, lateral, mortice

What is the most common ankle sprain? Lateral, anterior talofibular


ligament

Weber Type A (below ankle joint, syndesmosis intact), Type B (at ankle joint,
oblique, syndesmosis intact or slightly torn), Type C (above ankle joint,
syndesmosis disrupted, requires ORIF)
Ottawa Ankle/Foot Rules (Not for use in patients<18)
Ankle XR required if there is pain in malleolar zone, posterior edge 6cm
either side, or inability to weight bear immediately and in the ED
Foot XR required if there is pain in midfoot zone, base of 5th metatarsal,
navicular, inability to weight bear immediately and in the ED.

65yo M comes in compaining of knee pain


Ligament, meniscus, patellar dislocation, #, arthritis, bursitis, patellofemoral
syndrome, septic joint, ITB syndrome, medial plica syndrome, Osgood-Schlatter Dz,
osteochondritis dissecans, referred pain (hip vs sciatica)

1. HPI
Onset, trauma (contact, plant/pivot, jump, torque, lateral stress),
duration, severity, progression of pain, provoking/palliating
Morning stiffness <30min, night pain (infection)
Swelling/skin changes, instability
Crepitus/locking/catching (meniscal)
Fever, chills, night sweats, wt loss
Eye pain, rashes, dysuria, bloody stools, diarrhea, lower back pain
2. Meds/OTC/Allergies
3. PMHx/Surgery/Hospitalization
OA/RA, fracture, gout/renal dz
DM, IBD, bleeding d/o, cancer
4. Social Hx
Smoking/ETOH/IVDU
Travel (tick bites)
Function: bathing, cooking, clothing
Physical Exam
1. Inspection
Examine hands for nodes (Heberden's DIP and Bouchard's) and OA
Comments on Swelling, Erythema, Atrophy, Deformity
(varus/valgus/flat feet, alignment), Skin (color, warmth, scars,
lesions, wounds)
Gait, heel/toe walk, tandem walk, squat/duck walk
2. Palpation
Compare temperature, pulses, capillary refill, Baker's Cyst
Patella, tibial plateau, tibial tubercle, fibular head, femoral head,
tibia/femur.
3. ROM
Flexion/extension
Swipe test, apprehension test
Anterior/posterior drawer test (90deg), Lachman (30deg, sensitive),
pivot-shift test (ACL), valgus/varus stress
McMurray test (specific for meniscus, pain/click), Thompson test
Examines hip joint
4. Neuro
Tone, Motor (L2 hip flexion; L3 knee extension; L4 inversion of
foot; L5 extension of great toe; S1 eversion of foot)
Sensory (L2 lateral thigh; L3 middle knee; L4 middle ankle; L5
1st web space; S1 lateral foot) (light touch, pin-prick, vibration)
Reflexes (L4 patellar; S1 achilles, Babinski, clonus)

Ottawa Knee Rules

>55yrs

Isolated tenderness of the patella

Tenderness of the head of the fibula

Inability to flex to 90 deg

Inability to wt bear 4 steps at injury or at ED

65yo M comes in complaining of hip pain


OA, gout, RA, hip fracture, bursitis, femoroacetabular impingment, labral pathology,
ITB syndrome, inguinal hernia, UTI, kidney stone, referred pain from back/SI joint;
septic arthritis, osteomyelitis; meralgia paresthetica. Lyme dz, reactive arthritis,
psoriatic arthritis, sarcoid, trauma.

1. HPI
Onset, trauma, duration, location, severity, progression,
provoking/palliating
Morning stiffness <30min, night pain
Swelling, skin changes
Startup pain (bursitis), crepitus
Fever, chills, night sweats, wt loss,
Eye pain, rashes, dysuria, bloody stools, diarrhea, lower back pain
2. Meds/OTC/Allergies
Steroids, bisphophanates (proximal femur #)
BP pills (fall risk)
3. PMHx/Surgery/Hospitalization
DM, HTN, kidney dz (gout)
OA/RA, fractures
DDH as a child
IBD, TB
4. Social Hx
Smoking/ETOH/IVDU
Travel, tick bites
Function: bathing, cooking, clothing
Physical Examination
Hip Exam
1. Inspection
Examine hands for nodes (Heberden's DIP and Bouchard's) and OA
Comments on Swelling, Erythema, Atrophy, Deformity (pelvic tilt,
rotational deformity), Skin (warmth, scars, lesions, wounds)
Gait, heel/toe walk, tandem walk, squat
Trendelenburg, true/apparent leg length
2. Palpation
Anterior structures (iliac, trochanter, ASIS, inguinal ligament,
symphysis pubis, hip flexors, bursa)
Posterior structures (iliac, PSIS, ischial tuberosity, sacroiliac,
lumbosacral, coccyx)

3. ROM
Internal/external rotation (<15deg sensitive for OA)
Log-roll test (nondisplaced #),
FABER test (intraarticular hip, SI joint)
4. Neuro
Tone, Motor (L2 hip flexion; L3 knee extension; L4 inversion of
foot; L5 extension of great toe; S1 eversion of foot)
Sensory (L2 lateral thigh; L3 middle knee; L4 middle ankle; L5
1st web space; S1 lateral foot) (light touch, pin-prick, vibration,
proprioception)
Reflexes (L4 patellar; S1 achilles, Babinski, clonus)
5. Back Exam
Inspection, palpation, ROM, SLR
6. Counselling
Risk factors include: age, gender, obesity, occupation (on feet all
day), sports (wrestling, cycling, gymnastics, football). Running does
not appear to increase risk of OA
Cause unknown, can be secondary (DM, RA), gradual loss of
cartilage, worsens over time
Dx: symptoms and xrays
Tx: Rest, wt loss, VitD/Ca, heat/cold, PT, swimming/yoga/walking
Assistive devices (canes, walkers, raised toilets)
Tylenol (no more than 3g/day for chronic use), DO NOT drink ETOH.
For severe pain: T3s, NSAIDS, corticosteroid/hyaluronate injection
Very severe, not responding to treatment AND limiting function: OA
surgery - joint irrigation, joint fusion, joint replacement.
Offers educational pamphlets
PEP Questions

Name 4 findings on XR? Osteophytes, joint space narrowing, subchondral


cysts, subchondral sclerosis.

What investigations would you consider? CBC, lytes, Cr/BUN, ESR, CRP,
U/A and Cx, urine PCR for STI, serum urate

How would you treat a septic arthritis? Arthrocentesis daily, no NSAIDs, IV


abx x2 weeks, then PO x3-4wks

How would you treat gout? Colchicine (0.6mg BID) +/- steroids. If chronic:
VitC (500mg daily), then start allopurinol 2wks after acute attack to reduce
urate <360mmol.

What are the pathogens in septic arthritis? Gonococcal, S. aureus, S.


pneumo, H. influenzae, GNB, enterococcus, anaerobic

25yo F with low back pain. Do a focused history and physical.


Mechanical 90% (OA, muscle, spinal stenosis), disc herniation, radiculopathy,
sciatica (anterior thigh L4, lateral thigh L5, soles S1), cauda equina; AAA,
pyelonephritis; PID, ectopic, endometriosis; neoplastic, trauma, ankylosing
spondylitis, peptic ulcer, pancreatitis

1. HPI

Onset, duration, trauma, radiation, location, quality, severity,


provoking/palliating,
Morning stiffness (<30min), night pain (infection)
Bowel/urinary incontinence, leg paralysis/weakness, feeling
perineum, sexual function
Fever, chills, night sweats, wt loss,
Eye pain, rashes, other joint pain, dysuria, bloody stools, diarrhea
2. Meds/OTC/Allergies
Steroids, pain medication
3. PMHx/Surgery/Hospitalization
DM, HTN, AAA
OA/RA, ankylosing spondylitis, psoriasis,
4. Social Hx
Smoking/ETOH/IVDU,
Occupation, claims/compensation, time off, social supports
Function: sleep, exercises
Physical Exam
1. Inspection
Examine hands for nodes (Heberden's DIP and Bouchard's) and OA
Swelling, Erythema, Atrophy, Deformity (scoliosis, hyperlordosis),
Skin (warmth, scars, lesions, wounds, hair on lower back, cafe-aulait spots)
Gait, heel/toe, tandem walk, squat
Trendelenberg
2. Palpation
Spinous process, facet joints, paravertebral muscles, sacroiliac
joints, sacrum
3. ROM
Flexion, extension, side flexion, rotation, chest expansion
Schober's test (5cm below L5 and 10cm above, >30 cm ROM), SLR
(r/o disc herniation), cross-SLR (r/i disc herniation), Lasegue sign,
FABER (SI joint pain)
4. Neuro
Tone, Motor (L2 hip flexion; L3 knee extension; L4 inversion of
foot; L5 extension of great toe; S1 eversion of foot)
Sensory (L2 lateral thigh; L3 middle knee; L4 middle ankle; L5
1st web space; S1 lateral foot) (light touch, pin-prick, vibration,
proprioception)
(L4 patellar; S1 achilles, Babinski, clonus)
5. Counselling
Back pain is very common, most episodes resolve without tx (acute
resolves 90% w/in 1 month)
Continue regular activities modified
PT and exercise, supportive devices, massage, acupuncture
PEP Questions

What sx would make you worry about cauda equina? Saddle anesthesia,
anal tone decreased, fecal incontinence, urinary retention, bilateral lower
leg weakness.
If it is determined to be mechanical in nature, how long should one wait until
following up? 4 weeks.

75yo M comes in with facial drooping on the right side. Do a hx and px.
Ischemic (thrombosis, embolism, hypoperfusion), hemorrhagic (intracerebral vs
subarachnoid), TIA (<1hr). Etiology: atherosclerosis, atrial thrombus, endocarditis,
aortic dissection, MI, PE, tamponade; aneurysm rupture, HTN, trauma,
cocaine/amphetamines, vascular malformation (subarachnoid). Subdural, MS, tumor,
Bell's palsy, hypoglycemia, migraine, psychogenic
Triad: facial paresis/weak handgrip, arm drift, language (>80% stroke)

1. HPI
Onset (time), progression, previous episode
Headache, dysphagia/dysarthria/ataxia, weakness/numbness, visual
changes, vertigo, LOC
Fever, chills, cough, runny/stuffy nose
Head trauma, neck stiffness, N/V
Rashes, petechiae
CP/SOB, back pain, palpitations
2. Meds/OTC/Allergies
Anticoagulants, antihypertensive
OCP
3. PMHx/Surgery/Hospitalization
DM, HTN, dLp, stroke, CAD
Afib, aortic stenosis, bleeding disorder
MS, seizure, migraine
4. Social Hx
Smoking/ETOH/IVDU, cocaine, amphetamines
5. Family Hx
Stroke, aneurysm, coagulopathies
Physical Examination
Vital signs (compare both arms, irregular irregular)
1. Inspection
GCS, AAOx3, distress, pallor
Head trauma, posturing, nuchal rigidity
Petechial rash
Muscle bulk, fasciculations
2. CVS Exam
Capillary refill, carotid bruits
Auscultate precordium (endocarditis = murmur)
3. Neuro
Facial symmetry
Speech: You can't teach an old dog new tricks; dysarthria

4. Gait

CNII-XII
Finger-to-nose, repetitive movements, pronator drift
Elbow, hand supination, wrist tone (lead pipe/cog-wheel)
Tone, Motor/sensation/reflexes, Babinski, clonus
Romberg
Heel/toe walk, tandem walk

PEP Questions

Broca: non-fluent, understands; Wernicke: non-sensical, not understand

What is the most likely diagnosis? Bell's Palsy (CNVII: tasting, hearing,
muscles). DDX: Ramsay Hunt syndrome, pontine stroke, malignancy,
lymphoma, Lyme dz

At what blood pressure for an ischemic stroke should one be worried? SBP
>220, dBP >100

What would you do right away? CT non-contast head then give ASA 300mg
and thrombolysis (alteplase)

What would you use to decrease pressure? IV Labetolol

What is the window for using TPA? 4.5hrs


Rosier Scale
LOC (-1)
Seizure (-1)
Speech disturbance (1)
Visual field defect (1)

Unilateral leg weakness (1)


Unilateral face weakness (1)
Unilateral arm weakness (1)
Score >=1 likely stroke

35yo F, noticed thyroid englargement. Take hx and px


1. HPI
Onset, duration, progression, pain, fluctuation, hoarseness,
dysphagia, SOB
Hypo: cold intolerance, constipation, wt gain, weakness/fatigue,
depression, poor concentration, eye irritation, impaired hearing,
menorrhagia
Hyper: heat intolerance, sweating, diarrhea, wt loss, palpitations,
anxiety/irritability, oligomeneorrhea
Fever, chills, night sweats, cough, runny/stuffy nose, malaise (postinfectious)
Recently pregnant
2. Meds/OTC/Allergies
Levothyroxine, amiodarone, lithium, herbals
3. PMHx/Surgery/Hospitalization
Neck masses
Psychiatric hx
4. Social Hx
Smoking/ETOH/IVDU
Radiation exposure

Seaweed, caffeine,
5. Family Hx
Thyroid dz
Physical Examination
Vital signs (speed)
1. Inspection
Blank affect/anxiety, slow/rapid speech, body habitus
Dry/moist skin, hair loss
Onycholysis, clubbing, Tinnels test, fine tremor (paper test)
Pretibial myxedema
Periorbital puffiness, exophthalmos, ptosis
EOM (diploplia), lid lag
Tip head back (goiter, erythema, swelling, deviation)
Pemberton's sign (facial flushing, distended veins stridor, cyanosis)
Muscle strength (proximal), reflexes
2. Palpation
Check for adenopathy
Thyroid cartilage, cricothyroid membrane, cricoid, tracheal rings,
SCM
Isthmus around cricoid and tracheal rings, lobes run up behind SCM
Using finger pads, feel isthmus, get above and below, swallow water
Stabilize opposite side, palpate lobes (consistency, size, tenderness,
nodules, 15g normal weight)
3. Auscultation
Thyroid (bruits)
Hypothyroidism

Hyperthyroidism

Dry, cool, coarse skin


Wt gain, cold intolerance
Bradycardia
Periorbital puffiness
Swelling of face, hands, and legs
Myxedema
Dulled expression
Loss of lateral one-third of eyebrow

Moist, warm, smooth skin


Wt loss, heat intolerance/sweating
Tachycardia, palpitations
Exophtalmos, lid lag, stare
Onycholysis (detachment of nail)
Acropachy (swelling of digits with new
bone, Graves)
Nervousness

Primary: Hashimoto's, subacute


thyroiditis, post-partum thyroiditis,
lithium, amiodarone, iodine deficiency,

Pimary: Grave's dz, thyroiditis (subacute,


post-partum), toxic multinodular goiter,
toxic adenoma, amiodarone, levothyroxine,
seaweed, supplements.

Secondary: hypopituitarism (pituitary


adenoma), hypothalamic disorder

Secondary: TSH-secreting pituitary


adenoma, gestational thyrotoxicosis

Physical Exams (8 Stations)


BP Exam
1. Position
Seated, legs uncrossed, arm @ level of heart,
relaxed.
2. Cuff Choice/Placement
Bladder width 2/3 of arm circumference
2cm above antecubital fossa, snug, no clothing
3. Palpation
Radial artery, inflate cuff obliterated = systolic
(auscultatory gap)
4. Auscultation
Inflate cuff 20-30mmHg above palpation, deflate
2mmHg/sec
Listen for Korotokoff sounds, then disappear
5. Report
120/80, less than 140/90 which is hypertension

JVP Exam
1. Position
30-45deg, head to left,
expose neck, tangential
lighting
2. ID JVP
Landmark: SCM head to
mastoid, pulsation
Biphasic, occludable, nonpalpable, AJR, changes
with respiration/position
3. Measurement
Sternal angle, 90deg, 24cm
4. Report
high or low

Cardiac Exam
1. Inspection
Distress, LOC, pallor
Peripheral cyanosis, clubbing, capillary refill <3sec
Ankle edema, sacral edema
Central cyanosis, chest wall deformities/scars, heaves
Assess JVP
2. Palpation
Symmetry of radial pulses
Auscultate/palpate carotid artery (amplitude, rhythm, rate, volume)
Thrills/heaves, apical pulse (4-5th intercostal midclavicular: size,
location, duration), epigastric for RV impulse
3. Auscultation
AV/PV/TV/MV (S1/S2/S3/S4/murmurs)
Turn patient on left side/lean forward exhale/hold breath (mitral
stenosis/aortic regurgitation)
4. Respiratory Exam
Tracheal deviation
Anterior/posterior lung fields (resonant/dull)
Respiratory Exam
1. Inspection
Distress, LOC, pallor
Accessory muscle use, central cyanosis
Peripheral cyanosis, clubbing, nicotine staining, capillary refill <3sec

Calf tenderness/swelling, ankle edema


Symmetry of chest, intercostal indrawing, paradoxical indrawing
(modified Hoover's sign), chest wall deformities/scars, heaves
Barrel chested/hyperinflated
2. Palpation
Symmetry of radial pulses
Supraclavicular nodes, tracheal deviation, cricosternal height (<3-4
fingers = COPD)
Chest wall tenderness, excursion
3. Percussion
Anterior/posterior chest (resonant/dull, comparing sides)
Diaphragmatic excursion (3-5cm)
4. Auscultation
Anterior/posterior chest (vesicular, bronchovesicular/bronchial
sounds/crackles/wheezing)
5. Cardiac Exam (if patient has hemotypsis, do an abdominal exam)
Assess JVP
Auscultate/palpate carotid artery (amplitude, rhythm, rate, volume)
Thrill, fremitus, apical pulse (4-5th intercostal midclavicular: size
location, duration), epigastric for RV impulse
Auscultate: AV/PV/TV/MV (S1/S2/S3/S4/murmurs)
Turn patient on left side/lean forward exhale/hold breath (mitral
stenosis/aortic regurgitation)
At 9 min mark what would make you admit the patient?

CURB65: Confusion, BUN>7, RR>30, SBP<90 DBP<60, Age>65

What are signs of respiratory distress? Accessory muscle use, pardoxical


indrawing (modified Hoover's sign), intercostal indrawing, tracheal tug,
nasal flaring, pursed lip breathing, fragmented speech, tripoding, cyanosis.

DDX for hemotypsis: epistaxis, infection (TB, bronchitis, pneumonia),


cancer, alveolor (Wegener's, Goodpastures), PE, bronchial/pulmonary
artery hemorrhage/AVM, CHF, thrombocytopenia, coagulopathy, trauma.
R/O hematemesis

Abdominal Exam
1. Inspection
Distress/pain, AAOx3, overall jaundice/skin color/pallor
Palmar erythema, clubbing, contractures, nail changes (leukonychia,
koilonychia); asterixis
Scleral/frenular jaundice, fetor hepaticus, parotid hypertophy
Proximal muscle atrophy, hip and shoulder strength
Draping from xyphoid to pubic bone, examine from foot of bed
Gynecomastia, symmetry, scars, bruising, angiomata, caput
medusa, bulging flanks, pedal edema
2. Auscultation before palpation
Bowel sounds: high-pitched tinkling, bruits

Venous hum over liver


3. Percussion
Liver size (<12 cm, midclavicular line),
Castell's sign (last intercostal space, anterior axillary line),
Shifting dullness/fluid wave
4. Palpation, away from tender spot
Light/deep, rebound tenderness, McBurney's, Rovsing's sign,
Obturator/psoas (ROM of hip)
Liver edge (consistency), Murphy's sign, spleen (RLQ to LUQ), AAA
CVA tenderness,
DRE, testicular/pelvic exam
5. CV/RESP Exam
JVP, palpate nodes
Auscultate chest and heart
25yo M complains of right shoulder pain. Do a focused physical exam.
Supraspinatus tendinitis (impingement), rotator cuff tears, AC joint, subacromial
bursitis, bicep tendon rupture, osteoarthritis, frozen shoulder, fracture. PMR,
fibromyalgia, thoracic outlet syndrome, brachial plexus neuropathy, referred pain
(neck, gallbladder, heart, diaphragm, pancoast tumor)

1. Inspection: expose upper body


Swelling, Erythema, Atrophy (deltoid, trapezius, paravertebral),
Deformity (sling, clavicles), Skin (warmth, scars, lesions, wounds)
2. Palpation
Temperature, pulses
Sterno-clavicular joint, clavicle, acromioclavicular joint, coracoid,
head of humerus, greater tuberosity, spine of scapula
3. ROM
Hands behind head/up back, flexion, extension, abduction,
adduction, external rotation, internal rotation
Scapular winging test (anterior serratus)
Empty can test (supraspinatus), external rotation test
(infraspinatus/teres minor), deltoid abduction test, scap lift off
(subscapularis), painful arc test (40-120 degress, supraspinatus),
Hawking's test (rotator cuff impingment), Neer's test (impingment),
cross-arm test (acromioclavicular pain), apprehension test (anterior
instability), sulcus sign
Cervical ROM, Spurling's test
4. Neuro
Motor (C5 shoulder abduction; C6 elbow flexion; C7 elbow
extension; C6/7 wrist extension, wrist flexion; C7 finger
extension; C8 finger flexion; T1 finger abduction; T1 thumb
abduction
Sensory (light touch, pin-prick, vibration)
Reflexes (C5/6 biceps, brachioradialis; C7/8 triceps)

PEP Questions

Rotator cuff muscles (SITS) = Supraspinatus (abduction), Infraspinatus


(external rotation), Teres minor (external rotation), Subscapularis (internal
rotation)

25yo M fell off bike/MVA, with LOC +/- hit head. Do a physical exam.
Assess and Stabilize: airway, breathing, hemodynamic, neurologic status. Then
perform complete physical + neuro.

25yo M tackled playing football. Experiences acute CP. Do a px.


Tension PTX, massive hemothorax, open PTX, cardiac tamponade, flail chest. Rib
fracture, pulmonary contusion, PTX, blunt aortic injury, blunt myocardial injury,
diaphragmatic injury, esophageal perforation, arrythmia, MI,

25yo M was kicked in abdo. Abdo pain, has sore throat. Do a px.
LOC/GCS, asks about event, head injury, neck pain
1. Airway (A)
Talking, blood in mouth
2. Breathing (B)
Inspection: accessory muscle use, paradoxical indrawing, chest
symmetry, pallor, peripheral/central cyanosis
Palpate: tracheal deviation, neck hematoma, crepitus, broken
ribs/flail chest
Auscultate: breath sounds (vesicular, bronchial/crackles/wheezes),
symmetry, AE [R/O pneumothorax]
3. Circulation (C)
Vital signs, pulse both sides (rate, rhythm, quality)
Auscultate: heart (muffled heart sounds)
Bleeding, pelvic stability
4. Disability (D)
Smell of ETOH
Basilar #: raccoon eyes/battle sign/CSF
Cervical spine: palpate spine, ROM neck, parathesia in hands
PERL, motor, tone, posturing
5. Exposure (E)
Spinal tenderness, DRE (blood/tone/high prostate)
6a. Secondary Survey
CNII/III/IV/VI (EOM, visual fields, fundoscopy), CN5
(opthalmic/maxillary/mandibular, mastication muscles), CN7
(eyebrows, close eyes, show teeth), CN8 (hearing), CNIX/X ('ahh'
symmetry of palate), CNXI (shrug/head turn), CNXII (protude
tongue, side/side)
Tone, motor/sensation (touch, pain, temp)/reflexes, Babinski, clonus
Finger-to-nose, repetitive, pronator drift
Gait (heel/toe, tandem), Romberg
Palpates peripheral pulses
6b. Abdominal Exam
Bruising (Grey-Turner's/Cullen's sign), bulging flanks

Auscultates (BS, tinkling), aortic/renal bruits, venous hums,


Percuss liver, Castell's sign, shifting dullness, bladder distension,
Palpation: light/deep, rebound, liver edge, spleen, AAA
Testicular exam/pelvic exam
Management
Start IV fluids, stabilize vitals, start Abx
Consult general surgery, orthopedic, neurosurgery
PEP Question (a):

CBC, lytes, Cr/BUN, glucose, INR/PTT, type and screen and crossmatch,
AST, ALT, total bili, ALP, CK, lipase, ABG, tox screen, U/A.

ECG, lateral c-spine XR, CXR, pelvic XR for all patients

CT head for head injury, neuro deficits, intracranial HTN

CT chest for blunt trauma/penetrating, aortic injury

CT abdo/pelvis for penetrating/blunt trauma, peritoneal signs, hematuria

CT spine for mechanism, bony tenderness, neuro deficit.


PEP Questions (b):

Describe the CXR: pneumothorax with broken ribs

List 6 differential diagnoses? PTX, tension PTX, rib fracture, aortic


disection, esophageal rupture, tamoponade
PEP Questions (c):

Interpret CT scan that may or may not show splenic rupture

If patient in shock, requires 2 large bore IV, bolus NS, blood work,
crossmatch for blood, gen surg consult.
CT Head Rule
High Risk (for neuro sx)
Age >65
GCS <15 @2hrs post
Suspected skull fracture
Basal skull fracture
Vomiting >2 episodes
C-Spine Rule
High Risk
Age >65
Paresthesias
Dangerous mechanism (fall
from elevation >3ft/5stairs,
axial load to head, bicycle,
MVC >100km/h, rollover,
ejection)

Medium Risk (brain injury)


Amnesia before impact
Dangerous mechanism
(pedestrian, occupant ejected,
fall from elevation)

Rule Not Applicable


Non-trauma
GCS <13
Age <16
Coumadin or bleeding d/o
Obvious skull fracture

Any low-risk factors? YES


Simple rearend (exc. truck/bus)
Ambulatory at any time
Sitting position in ED
Delayed onset neck pain
No midline tenderness

If NO, then radiograph

Able to rotate neck? YES then


NO RADIOGRAPHY

Rule Not Applicable


Non-trauma
GCS <15
Unstable
Age <16
Acute paralysis
C-spine issues

60yo M with sx of Parkinsons/balance. Do a focused physical exam.


1. General (orthostatic vitals)
Expressionless, excessive sweating, decreased blinking, soft voice,
poverty of movement, drooling
Resting tremor (pill rolling, slow 5hz, PD=asymmetric)
AAOx3, repeat these 3 words, spell WORLD/backwards, no ifs
ands or buts, what is this x3, repeat those 3 words

2. Neuro
CNII-XII, EOM: nystagmus, impairment of upward gaze, delayed
initiation, slowing of conjugate movement
Glabellar tap (PD can't suppress) [everything else normal]
Finger-to-nose, repetitive movements, pronator drift
Elbow, hand supination, wrist tone (lead pipe/cog-wheel)
Tone, motor/sensation/reflex, Babinski, clonus
3. Gait
Rise, walk, turn around, sit (stooped, narrow stance, shuffling step,
decreased arm swing, festination)
Romberg (not affected by PD)
PEP Questions

PD is a clinical diagnosis (R/O wilson's dz and hyperthyroidism)


Cranial Nerve Exam
General
I (olfactory)
II (optic)

Facial asymmetry, pupil asymmetry, ptosis,


Ask about any changes in smell
Visual acuity: Snellen chart, Visual fields: Finger test
Light reflexes: direct, consensual, swinging torch
Accomodation; Fundoscopy
III, IV, VI (oculomotor,
Ask if diploplia ever occurs
trochlear, abducens)
H Pattern testing: nystagmus, dysconjugate gaze (MLF)
V (trigeminal)
Pin prick/light touch (V1,V2,V3), Corneal reflex
Sensation and motor
Clench teeth, and palpate masseter/temporalis,
Open jaw (test pterygoid)
VII (facial) Anterior 2/3
Any change in taste?
tongue for taste
Forehead wrinkling, eye closure, blowing of cheeks, smiling
VIII (vestibulocochlear)
Otoscopy. Whisper test. Rinne and Weber.
IX and X
Ask about dysphonia and swallowing. Cough.
(glossopharyngeal, vagus) Uvula deviation (ahhh). Gag reflex.
XI (accessory)
Shoulder shrug and head turning.
XII (hypoglossal)
Tongue deviation, wasting fasciculations
Assess tongue strength against cheek.
UMN
LMN
Power
UE flex > extend
Decreased/absent
LE extend > flex
Tone
Spastic
Flacid
DTRs
Increased
Decreased/absent
Plantars
Upgoing
Downgoing
Atrophy
Late
Early

50yr F in for breast exam. Teach her to do one.


Today I have been asked to teach you a breast exam
Would you like a chaperone.
Best time is 1 week after period.
1. Inspection
Look in mirror, scars, asymmetry (often normal), masses
Skin changes: erythema, puckering, skin dimpling (peau d'orange)

Nipple changes: retraction, rash, discharge, eczema (Paget's)


Repeat: arms above head, and hands on hips. Look for tethering
2. Palpation
Lie down flat, place pillow on side of examination
Use the flat of fingers in spirals around breast. Compress against
chest wall.
Note for lumps
Ensure palpation of axillary tail, palpate nipples too.
Looking for texture, tenderness, mobility.
3. Lymph nodes
Examine axilla for adenopathy, palpate all sides
Repeat other side
Palpate cervical, supraclavicular and infraclaviular
4. Summarize
Mammography recommendations @age 50 or 10 years before
family hx

History Taking (29 Stations)


55yo M going in for surgery. Take a focused history.
What surgery, emergent/elective
1. HPI
CP/SOB, PND, orthopnea, weight gain,
Climbing stairs/walking, how many blocks
Snoring, sleep, refreshed
2. Meds/OTC/Allergies
Anticoagulants, nitrates, beta-blockers, ACEi, ARB, diuretics
Hypoglycemics, insulin, steroids
Allergies to latex and anaesthetics
3. PMHx/Surgery/Hospitalization
DM, HTN, heart dz, stroke, CHF, arrythmia, aortic stenosis
Asthma/COPD, OSA(CPAP), GERD
CKD, anemia, coagulopathy
Spinal cord lesion, neuromuscular dz, seizures
Rheumatoid, neck arthritis/injury
4. Social Hx
Smoking/ETOH/IVDU
5. Family Hx
Malignant hyperthermia, cholinesterase deficiency Dz
Revised Cardiac Risk Score
1. History of ischemic heart disease
2. History of congestive heart failure
3. History of cerebrovascular disease (stroke or transient ischemic attack)
4. History of diabetes requiring preoperative insulin use
5. Chronic kidney disease (creatinine > 2 mg/dL)
6. Undergoing suprainguinal vascular, intraperitoneal, intrathoracic surgery
Risk for cardiac death, nonfatal MI/cardiac arrest
0 pt = 0.4%, 1 pt = 0.9%, 2 pt = 6.6%, 3 pt = >11%

50yo M has chest pain, take a history.


Stable/unstable angina, NSTEMI/STEMI, aortic dissection, PE, PTX/Tension,
esophageal rupture/spasm, pneumonia, pleuritis, costochondritis, herpes zoster, rib
fracture.

1. HPI
Onset, duration, previous episode, progression, location, radiation,
quality (tearing/sharp/crushing), severity, provoking/palliating
(exertion, resting, eating, position)
Diaphoresis, pallor, N/V, palpitations, syncope
Fever, chills, night sweats, cough (blood), SOB, runny/stuffy nose
(pericarditis, myocarditis)
Immobilization/calf swelling (PE), trauma (PTX, dissection)

Heart burn/dysphagia, decrease in physical activity (costochondritis)


2. Meds/OTC/Allergies
Warfarin, heparin, ASA, clopidogrel
3. PMHx/Surgery/Hospitalization
DM, HTN, lipids, MI/angina, valvular dz, clotting disorder
Asthma/COPD, GERD/PUD
Anxiety/panic attack
CABG, pacer, valves
4. Social Hx
Smoking/ETOH/IVDU, cocaine
Travel hx, immobilization
5. Family Hx
Heart disease
PEP Questions:

What are 5 important risk factors to ask this patient? Male >45, F>55; DM,
HTN, high cholesterol/low HDL, elevated inflammatory biomarkers, obesity,
smoking, FAMhx <60

What other 4 diagnostic tests would you order? ECG, troponin, CXR,
echocardiogram, Lytes/coagulation/hemoglobin
CS Angina Classification:

I prolonged activity

II with strenuous activity

III daily activity

IV at rest/unstable

Aortic Dissection:

Type A: ascending/arch
(surgical)

Type B: does not involved


acending/arch

75 F has SOB, take a history.


CHF, anemia, PE, PTX, asthma/COPD, deconditioning, panic attack. Pneumonia,
pleural effusion, interstitial lung dz, lung cancer. MI, tamponade, anaphylaxis,
myasthenia gravis.

ABCs
1. HPI
Onset, duration, previous episodes, provoke/palliating (rest/exertion)
Blocks/stairs, orthopnea/PND (#pillows), CP (OPQRSTPP)
Fever, chills, night sweats, cough/sputum/blood, runny/stuffy nose
Leg swelling/calf pain, pregnancy
Bloody stools, bleeding, fatigue, lightheadedness, syncope
Nutrition, appetite, daily food
2. Meds/OTC/Allergies
OCPs, heparin
Nitrofurantoin, amiodarone, bleomycin, methotrexate
Flu/pneumococcal vaccine
3. PMHx/Surgery/Hospitalization
CHF, asthma/COPD, PE/DVT, anemia
Anxiety/panic attacks

4. Social Hx
Smoking/ETOH/IVDU(talc lung),
Occupation (asbestos), sick contacts
Travel/sexual hx
5. Family Hx
Cancer, heart disease, CF
PEP Questions:

Turns out she eats on tea and toast, likely anemic.

What would you see on a peripheral blood smear? Megaloblasts due to B12
deficiency.
NYHA Classification

I Cardiac disease but no symptoms

II Mild SOB and slight limitation in ordinary activities

III Marked limitations, comfortable only at rest

IV Severe limitations, experiences symptoms at rest


PERC Rules (If any positive cant r/o PE)
Age>50, HR>100, SpO2 <95%, prior DVT/PE, recent trauma/surgery, hemoptysis,
estrogen, unilateral leg swelling.
Wells Criteria for PE
Sx of DVT [+3], PE is likely [+3], HR>100 [+1.5], immobilization or sx [+1.5],
previous PE/DVT [+1.5], hemoptysis [+1], malignancy [+1]
Wells Criteria for DVT (1 point each)
Cancer, bedridden/surgery, calf swelling >3cm, collateral veins in leg, leg swollen,
tenderness along deep venous, pitting edema, immobilization extremity, previous
DVT, alternative more likely [-2]: >2 high risk

25yo M comes in with history of cough. Take a history.


Acute (3wks): acute bronchitis, pneumonia, asthma/COPD. Subacute (3-8wks): postinfectious, pertussis, asthma, sinusitis. Chronic (>8wks): chronic bronchitis,
postnasal drip, post-infectious, GERD, foreign body, tumor, smoker's cough. PE,
pneumonia, CHF, cancer, TB.

1. HPI
Onset, duration, night symptoms, productive (color/blood),
provoking/palliating (meals, position, smoke, pets, exercise)
Barking, wheezing, drooling, sore throat, hoarseness
CP/SOB
Runny/stuffy nose, facial pain/ear pressure
Fever, chills, night sweats, wt loss
Calf tenderness/swelling
Hearburn, waking up with metal taste
Myalgia/rashes (mycoplasma)
2. Meds/OTC/Allergies
Inhalers, steroids, antibiotics,
ACEi, beta-blockers,

Immunizations
3. PMHx/Surgery/Hospitalization
Asthma/COPD, PE/DVT
CHF, GERD
TB/HIV, CF, sarcoidosis
Pneumonia, sinusitis
4. Social Hx
Smoking/ETOH/IVDU, pets
Travel/sexual hx (STD)
5. Family Hx
Asthma, eczema, lung dz
PEP Questions:
CXR, loculated opacity in RUL. What test would you order? AFB sputum,
sputum culture, thick and thin smear, needle aspiration or biopsy >
transbronchial. DDX: TB, granulomatous disease (sarcoid, Wegeners),
malignancy, fungal infection.
Common cold (rhinovirus 50%, coronavirus 15%, influenza 15%, unknown
20%),
Acute pharyngitis (viral 90%, GAS, beta-hemolytics strep, N. Gonorrhea),
Sinusitis (viral, S. pneumoniae, H. influenza, M. catarrhalis, anaerobes if
chronic).
If asthmatic, PFTs: ventolin (250mg 2 puffs BID), fluticasone (125mg 2 puffs
BID), ipratropium.
Community acquired (S. pneumoniae, M. pneumoniae, C. pneumoniae,
Chlamydia, H. influenzae, M. catarrhalis, S. aureus, viral)

65yo M with abdominal pain/jaundice. Do a hx.


Based on location: gastroenteritis, biliary, hepatitis (infection, autoimmune,
hemachromatosis, Wilson's, drugs), pancreatitis, appendicitis, diverticulitis,
GERD/gastritis, esophagitis, obstruction. PBC/PSC, IBD, sigmoid volvulus, abscess
(subdiaphragmatic, splenic), mesenteric adenitis, salpingitis, pericarditis, porphyria,
lead poisoning

1. HPI
Onset, duration, timing, night pain, severity, quality (burning, tearing,
colicky), radiation (shoulder pain, R subscap, back),
provoking/palliating (eating, BM, urination, position)
N/V, diarrhea, constipation, last bowel movement, bloody stools
Fever, chills, night sweats, wt loss/appetite, abdominal mass
Jaundice, pruritus, confusion, dark urine/pale stools
Cough/blood, bruising, waist size, ankle swelling
Joint pain, rashes, skin bronzing
2. Meds/OTC/Allergies
Tylenol, OCPs
3. PMHx/Surgery/Hospitalization
Liver/gallbladder dz, cancer
IBD, RA, autoimmune

Blood transfusions
4. Social Hx
Smoking/ETOH/IVDU, tattoo
Travel/sexual hx (STD)
Occupation (HCW)
5. Family Hx
IBD, cancer, hemachromatosis
PEP Questions

What can you see on AXR? Bowel obstruction, volvulus, pneumatosis,


biliary tree air, calcification, colitis

What blood tests would you consider? CBC, lytes, BUN/Cr, glucose, betahcg, AST, ALT, ALP, bili, lipase, U/A, culture

What would US show? AAA, gallbladder, renal dz, pancreatitis, venous


thrombosis, peritonitis, pelvic dz, indicated for pregnancy.

Primary billiary cirrhosis (PBC) - autoimmune dz; destruction of intra hepatic


bile ducts; seen mainly in women. Associated with RA , Sjogren. AMA found
in 95%.

Primary sclerosis cholangitis (PSC) focal dz; destruction of intrahepatic


and extrahepatic ducts; seen mainly in males. Associated with IBD. ERCP
see characteristic strictures.

Acute abdominal pain and jaundice. List the 5 most important questions to
ask in your history: IVDU, ETOH, blood transfusion, tattoos, asian ethnicity.

What is your differential diagnosis? Hepatocellular, cholestatic, CHF,


nephrotic sx, malignancy.

What investigations would you order in this patient? CBC, LFTs, lytes, abdo
U/S, CT, liver biopsy, diagnostic paracentesis

35yo M comes in with melena/bloody stools. Take a focused history.


Painful - PUD, IBD, fissure, Mallory-Weiss tear, gastritis, esophagitis, hemorrhoids.
Painless polyps/cancer, diverticulosis, esophageal varices, AVM.

1. HPI
Onset, duration, previous episode, color (red/tarry), volume of blood,
on wiping/stool/toilet bowl, abdominal pain, straining, constipation,
diarrhea, stool changes, last bowel movement
Acid reflux, dysphagia, cough, SOB/CP
Jaundice, ascites, edema (esophageal varices)
Nose bleed, bleeding gums, bruising, petechiae
Fever, chills, night sweats, wt loss
Rashes, joint pain, dry eyes, dry mouth
Fatigue, tachycardia, light-headedess
2. Meds/OTC/Allergies
NSAIDs, anticoagulants, steroids,
Pepto-Bismol, beets, iron supplements
3. PMHx/Surgery/Hospitalization
GERD/PUD, IBD, liver dz, hemmorhoids
Transfusions
4. Social Hx

Smoking/ETOH/IVDU
Travel/sexual Hx
5. Family Hx
IBD, colorectal cancer, other cancer
PEP Question

How do you treat H. Pylori? Clarithromycin 500mg BID and metronidazole


500mg BID and omeprazole 20mg BID x14days

How do you R/O UGIB? NG aspirate

Endoscopy (sigmoidoscopy/colonoscopy), Gastroscopy (upper GI bleed)

Non-variceal bleed: bolus 80mg IV pantoprazole, then run 8mg/hr x72hrs

If variceal bleed: 50mcg IV octreotide, then drip 50mcg/hr

45yo M comes in with a history of diarrhea. Take a history.


IBD/IBS, celiac, cancer, lactose intolerance, hyperthhyroidism, Addison's, uremia,
pancreatitis, carcinoid, infection.

1. HPI
Onset, duration, frequency, quantity, quality (bloody/tarry,
watery/mucous, foul smelling/floats), urgency (rectal),
provoking/palliating (fasting = secretory, wheat, milk)
N/V, abdominal pain, bloating, gas
Fever, chills night sweats, wt loss
Sore throat, cough
Joint pain, rashes, red eyes, mouth ulcers
Palpitation, heat intolerance, sweating
Food (poultry, spoiled dairy, hamburger, seafood),
2. Meds/OTC/Allergies
Abx, chemotherapy, laxatives
Metformin, colchicine
3. PMHx/Surgery/Hospitalization
IBD/IBS, celiac
Hyperthyroidism, Addison's,
Short-gut syndrome (previous surgery)
4. Social Hx
Smoking/ETOH/IVDU
Travel/sexual hx
Occupation/HCW, childcare
5. FamHx
Colon cancer, IBD, celiac dz
PEP Question

What special tests would you consider? Anti-TTG, IgA levels. TSH
Organisms: small bowel/watery (salmonella, cholera, C. diff, campylobacter,
yersinia), colonic/bloody (shigella, EHEC, Campylobacter, Yersinia).
Noro/rotavirus. Cryptosporidium, Giardia.

35yo F comes in complaining of vomiting. Take a history.


Gastroenteritis, obstruction, appendicitis, pancreatitis, GERD. Pregnancy, meningitis,
vertigo, DKA, adrenal insufficiency, migraine, uremia, hyperthyroidism. Bulimia,
anxiety, depression, MI, HTN emergency.

1. HPI
Onset, duration, frequency, content (blood, coffee grounds, food,
bilious), amount, projectile vomiting, provoking/palliating factors
(meals, food, time, lying)
Abdominal pain (PQRSTPP), constipation/diarrhea, last bowel
movement, flatus, last ate/drank
Fever, chills, night sweats, wt loss
Headaches, vertigo, photophobia, neck stiffness
Heat intolerance, sweating, palpitations
LMP, pregnancy
Stressors, mood, suicide, anxiety
Anorexia, guilt, early satiety
Polyuria/decrease (uremia), postural hypotension, CP/SOB
2. Meds
Opioids, steroids, NSAIDs
3. PMHx/Surgery/Hospitalization
DM, pancreatitis, IBD
Hernias, colon cancer
Depression, anxiety
4. Social Hx
Smoking/ETOH/IVDU,
Occupation
Note:

Go into room, withdrawn pale woman. Vague history of abdo pain and
vomiting, non specific.
She is depressed/adjustment disorder. Her father recently died.

25yo F, first prenatal visit/GA36wks with BP150/90 or GDM. Do a hx.


Fetal movements, pre-eclampsia, bleeding, cramping.

1. HPI

GTPAL, planned/unplanned, LMP


Breast tenderness, N/V, urinary frequency, wt gain
Fetal movements, cramping, discharge/bleeding
Headaches/drowsiness, blurred/double vision/amourosis/flashing
lights, SOB/CP, abdo pain, edema, urine output
U/S: dating (10-13wks), detailed (18-20wks)
Prenatal care: HTN, GDM, infection, CBC
2. Meds/OTC/Allergies
Contraception, vitamins (iron/folic acid)
Anti-hypertensives
3. PMHx/Surgery/Hospitalization

HTN, DM, kidney dz


HIV, chicken pox
Previous pregnancy: GDM/HTN, infection, bleeding, fever/GBS, UTI,
GA/delivery, NICU, metabolic screen, complication
(fever/shock/seizure/distress), meconium.
Health of child now
Last PAP smear
4. Social Hx
Smoking/ETOH/IVDU
STD history
Occupation/husband, support system
Asks about delivery, if severe pre-eclampsia->hospital->Obs consult.
5. Family Hx
Genetic conditions, CF
6. Counselling
1st Trimester (wk1-12)
2nd Trimester (wk13-28)
Detailed US at 18-20wks.
Regular monthly visits to week 28
Smoking/ETOH/drugs, folic acid, iron,
Subsequent visits 24-28wks:
support system.
CBC, GDM screen, (24-28wk)
Initial Labs: beta-hcg, CBC, blood
blood type and antibodies
type/Rh, rubella, varicella, HBsAg,
VDRL, HIV, gonorrhea/chlamydia
3rd Trimester (wk29-40)
screen, urine dip
GBS swab at 36wks
Dating U/S 10-12 wks
Women >40yrs: CVS 10-12wks (1-2%
SA) , Amnio 15-16wks (0.5% SA)
Screening
Explain DS: Tri21, genetic defect, causes mental impairment and physical
abnormalities. Each person different. No cure, but resources available.
Screening = risk, not diagnostic. E.g. 1/100
Detection rate of ~85% and false positive rate of <5%
Explains risk with prenatal diagnosis (SA to fetus, other
abnormalities found, no clear results found.)
Will they consider abortion? Explains psychological implications of
prenatal screening
SIPS is 2 parts (10-14wks and 15-21wks). If >35y can get IPS
(includes NT), If >40 = amniocentesis. NIPT which is private pay
(>10wks). Next step is diagnostic (CVS, amnio)
SIPS (MSP)

IPS (>35)

FTS (varies)

Amnio (>40)

NIPT ($800)

Tr21/18, NTD

Tr21/18, NTD

Tr21/18/13

All

Tri21/18/13++

2 draws

2 draws+1 US

1 draw+1 US

Invasive

1 draw

10-14+15-21w

10-14+15-21w

11-14w

>15w

>10w

DR >85%

87%

83%

100%

99.9%

FP 4.4%

1.9%

5.0%

0%

0.2%

No risk

No risk

No risk

1/200 SA

No risk

PEP Questions:
What was most concerning? Visual changes, headache, abdominal pain,
Severe pre-eclampsia: cerebral/visual distrubance, hepatic (abdo pain,
LFTs), BP>160/110, low PLT, renal abnormality, pulmonary edema.
How would you dx? SBP> 140/90 AND Proteinuria >0.3g/24h or signs of
end-organ dysfunction (PLT, Cr, AST, ALT)
Complication? Seizures, growth restriction, preterm delivery, abruptio
placentae, stillbirth.
3 Procedures/Investigations? US, BPP, BW
Medication? Labetolol or Hydralazine or PO Nifedipine AND MgSO4
Sugar in urine, need to do more tests. Risk of diabetes in the future. Risk to
fetus include macrosomia, shoulder dystocia, requires surgery. Reassure
mother.

25yo F comes in complaining of missing her periods. Take a history


Pregnancy, anorexia/stress/exercise, prolactinoma/hypothalamic tumor, Kallman's
syndrome, Sheehan's vs. PCOS, menopause/premature ovarian failure (Turner's,
radiation, chemotherapy), Ashermans, hypothyroidism, Cushing's

1. HPI
Onset, duration, previous episodes, LMP (menarche, regularity,
length of cycle/menses, #pads), dysmenorrhea
Breast tenderness, N/V, bloating
Hot flushes, night sweats, vaginal dryness, mood changes
Diet, wt loss, exercise, stress
Wt gain, cold/heat intolerance, dry/wet skin, sleeping
Headache, visual changes, galactorrhea, smelling problems
Hirsutism, balding, acne
2. Meds/OTC/Allergies
OCP/IUD, anti-psychotics
3. PMHx/Surgery/Hospitalization
Pregnancy/infertility, hemorrhage
Breast feeding/failure
Cancer treatment
Abortion, D&C
4. Social Hx
Smoking/ETOH/IVDU
Sexual Hx (#/concurrent, sex for money/drugs, oral/vaginal/anal)
STD hx
Stressors (relationship, school, work)

5. Family Hx
PCOS, DM, thyroid dz
PEP Questions

What blood work would you order? B-HCG, FSH, LH estradiol, testosterone,
androstenedione, DHEA

What imaging would you do? U/S transvaginal, MRI head, if suspicion:
hysterosalpingogram, hysteroscopy

What other tests would you consider doing? Progesterone challenge test,
karyotype, 24h-urine cortisol

25yo F who is pregnant. Experienced vaginal bleeding. Take a history.


1st trimester (Nonviable: SA [20%], ectopic, molar; Viable: normal implantation [33%],
cervical polyp, rectum/bladder). 2/3rd trimester: (placenta previa [painless], vasa
previa, abruption [painful, dark blood], bloody show, cervical/vaginal lesion)

1. HPI
Onset, duration, trauma, previous bleeding, color (dark = abruption),
underwear/wiping/urine, frequency of changing pad/tampon,
passage of tissue/clots, post-coital bleeding
Fetal movements
Cramping/contractions, back pain
LMP, U/S results (ectopic/twins)
Prenatal care (BP, blood type)
2. Meds/OTC/Allergies
OCPs, IUD, anticoagulants
3. PMHX/Surgeries
Bleeding disorder, liver dz
Previous C/S
Previous pregnancy: GDM/HTN, infection, bleeding, fever/GBS, UTI,
GA/delivery, NICU, metabolic screen, complication
(fever/shock/seizure/distress), meconium.
Health of child now
Last PAP smear
4. Social Hx
Smoking/ETOH/IVDU, cocaine
STD hx
Support at home, occuptation/husband, plans for pregnancy
PEP Questions
Placenta previa RF: grand multip, twins, age>35, uterine scar - Tx:
cerclage.
Placenta abruption RF: previous abruption, HTN/preeclampsia, trauma,
PROM, smoking, cocaine.
How would you manage the bleeding? ABC, Rhogam, transvaginal U/S,
CBC, serial betaHCG
How would you manage ectopic? Stable = methotrexate (betaHCG
decrease 15%/week), unstable = laparoscopy. Perform U/S at 7 weeks to
ensure no recurrence (10%)

In normal pregnancies beta-HCG doubles q48-72 hours until 10,00020,000mIU/ml. At betaHCG of 1500-1800mIU/ml (transvaginal U/S), with
abdo U/S (6000-6500mIU/mL)
Betamethasone x2 if GA <32wks, consider emergent delivery if: maternal
instability, placental insufficiency, IUGR, preeclampsia, non-reassuring FHR

25yo F has experienced vaginal bleeding/menorrhagia. Take a history.


DUB (due to anovulation, unopposed estrogen, exogenous hormone), PCOS,
pregnancy, fibroids/polyps, thyroid dysfunction, atrophic vaginitis, GU neoplasm.
Menorrhagia (regular cycle >7days, >1pad q2hrs), metrorrhagia (irregular, between
cycles), menometrorrhagia (metrorrhagia that is prolonged), polymenorrhea (<21d
cycle), oligomenorrhea (>35d cycle),

1. HPI
Onset, duration, previous episode, LMP (menarche, regularity,
length of cycle/menses), frequency of changing pad/tampon (q1-2
hours too much), color, passage of tissue/clots, post-coital bleeding
Fever, chills, night sweats, pain, discharge
Abdominal mass, bloating, early satiety
Breast tenderness, N/V, urinary frequency
Cold/heat intolerance, wt gain/loss, palpitations
Wt gain, hirsutism, acne
2. Meds/OTC/Allergies
OCP/IUDs
Anticoagulants
3. PMHx/Surgery/Hospitalization
Fibroids, PCOS
Thyroid dz, bleeding disorder
PAP smear results, previous cancer
4. Social Hx
Smoking/ETOH/IVDU
STI Hx
5. Family Hx
Cancer
PEP Questions:

What for ovulatory menorrhagia? NSAIDS (onset, q6-8hrs PRN)

What surgical procedure? D&C, endometrial ablation, hysterectomy

55yo F comes in complaining of dysuria. Take a history.


UTI/cystitis, urethritis, prostatisis, epididymitis, orchitis, pyelonephritis

1. HPI
Onset, duration, previous episodes, pain (PQRSTPP), frequency,
discharge, urine color, does urinating make the pain go away
Flank/back pain, abdo fullness, incontinence
Fever, chills, night sweats, N/V
Urethral/vaginal discharge, joint pain, skin rash, lesion/swelling
LMP (menopause)

2. Meds/OTC/Allergies
Abx, contraception
Anticholinergics (antihistamines, antidepressants)
3. PMHx/Surgery/Hospitalization
Prolapse, GTPAL
Menopause
DM, kidney stones, BPH, incontinence
4. Social Hx
Smoking/ETOH/IVDU
Sexual Hx (#/concurrent, sex for money/drugs, oral/vaginal/anal)
STD hx
5. FamHx
Cancer
PEP Questions

UTI - KEEPSS: Klebsiella, E.coli [95%], Enterococcus faecalis, Proteus


mirabilis, Pseudomonas, Staph saprophyticus, Serratia marcesence;
Urethritis: gonorrhea, chlamydia, trichomonas vaginalis, HSV

What is the treatment for cystitis? Septra DS 1tab PO BID x3d OR


Nitrofurantoin 100mg PO bid x5d. If male Septra DS 1tab PO bid x7d

What is the treatment for recurrent cystitis? Low dose or post-coital Abx:
Septra 1tab PO daily, or post-coital only.

Do you treat asymptomatic bacteruria? Only if pregnant

How would you treat prostatitis? Cipro 500mg PO q12h x2-4wks or Septra 1
tab PO BID x2-4wk

25yo F with lower abdominal/pelvic pain. Take a focused history.


Dysmenorrhea, ectopic, PID, endometriosis, fibroids, ovarian
torsion/hemorrhage/rupture, prolapse, UTI vs. appendicitis, diverticulitis, obstruction,
renal stone, ruptured AAA, mesenteric ischemia, IBD, cancer, hernia, hip arthritis,
shingles.

1. HPI
Onset, duration, pattern (cyclical), progression, location, quality,
radiation, provoking/palliating (BM/voiding, menses, sex, NSAIDs)
Fever, chills, night sweats, wt loss
Discharge, itchiness, irritation, odor, bleeding
Dysuria, back pain
LMP (regularity, length cycle/menses), breast tenderness, N/V,
cramping, bloating, pregnancy test
Last bowel movement, diarrhea/constipation, bloody stools
2. Meds/OTC/Allergies
OCP/IUD
3. PMHx/Surgery/Hospitalization
PID, previous pregnancy, infertility, PAP
Appendicitis, IBD
4. Social Hx
Smoking/ETOH/IVDU

Sexual Hx (#/concurrent, sex for money/drugs, oral/vaginal/anal)


STD hx
5. Family Hx
Fibroids, endometriosis
Breast, ovarian, colon, endrometrial cancer
PEP Questions:

Ectopic vs. PID, she was having periods so PID.

Antibiotics for inpatient? Cefoxitin (2g IV q6h, D/C after 24hr clinical
improvement) and doxycycline (100mg PO BID x14days)

Antibiotics for outpatient? Ceftriaxone (250mg IMx1) plus doxycycline


(100mg BID 14 days) plus metronidazole (500mg BID 14 days)

Indications for hospital? Pregnancy, non-adherence, N/V, fever, pelvic


abscess.

25yo F comes in complaining of vaginal discharge. Take a history.


UTI vs Vaginal infection: bacterial vaginosis, candidiasis, trichomoniasis, atrophic
vaginitis [R/O PID]

1. HPI
Onset, duration, previous episode, quantity, frequency, color,
consistency (milky:vaginosis, thick:candidiasis, frothy:trichomonas),
odor (fishy:vaginosis), provoking/palliating (menses/sexual activity)
Pain, itchiness, irritation, lesions
Fever, chills, night sweats, abdominal pain
LMP (menarche, regularity, length of cycle/menses)
Dysuria, urinary frequency, nocturia, smelly urine
Douching, foreign body
2. Meds/OTC/Allergies
OCP/IUD
Antiobiotics, immunosuppressants
3. PMHx/Surgery/Hospitalization
DM, HIV
Pregnancy, PAP smear
4. Social Hx
Smoking/ETOH/IVDU
Sexual Hx (#/concurrent, sex for money/drugs, oral/vaginal/anal),
STD hx
PEP Questions:

On a pelvic exam what would you look for? Vulvar erythema, atrophy,
lesions, cervix friability.

What tests would you do? pH, whiff test, wet-mount, gram-stain, GC screen.
Consider VDRL, HIV, HepB

BV: metronidazole 500mg PO bid x7d or clindamycin cream;

Trichomoniais: metronidazole 2g PO x1dose

Candidiasis: fluconazole 150mg PO x1dose

Gonorrhea: cefexime 800mg PO x1dose or ceftriaxone 250mg IM x1

Chlamydia: doxycycline 100mg PO BID x7d or azithromycin 1g PO x1

75yo F has fatigue/anemia. Take a history.


Anemia (normocytic: blood loss, HIV, malignancy, autoimmune, chronic infection,
early Fe deficiency, hypothyoidism, alcoholism, liver dz, sickle cell, chronic renal dz,
hemolysis; Microcytic (TAILS): thalassemia, anemia of chronic dz, Fe deficiency, lead
poisining, sideroblastic anemia; Macrocytic: B12/folate deficiency, chronic liver dz,
AIHA, myelodysplasia, pernicious anemia, chemotherapy, antiviral meds,
hypothyroidism.
CHF, hypothyroidism, depression, CFS, OSA, DM, substance abuse, cancer, HIV,
medication/toxin.

1. HPI
Onset, duration, progression, timing, weakness/pain,
provoking/palliating factors (exertion, rest)
Bloody stools, melena, bloody urine, lightheadedness/syncope
Nutrition/appetite, dysphagia, dental health
Fevers, chills, night sweats, wt loss
Headaches, weakness, dysarthria, ataxia, visual changes, vertigo
CP, palpitations, orthopnea/PND, abdo swelling, edema
SOB, wheeze, cough, blood
Cold intolerance, dry skin, constipation
Stressors, mood, concentration
Sleep, snoring, refreshed
2. Meds/OTC/Allergies
Beta-blockers, ACEi, diuretics
Antidepressants
3. PMHx/Surgery/Hospitalization
DM, CAD, COPD,
Sleep disorder, OSA
Hypothyroidism, cancer
Depression, anxiety
4. Social Hx
Smoking/ETOH/IVDU
Travel Hx, tick bites
STD Hx
5. FamHx
Cancer
PEP Questions:

What labs would you order: CBC, retic count, peripheral smear
(schistocytes MAHA, target cell liver dz; hypersegmented neutrophils
b12; tear rop thalassemia; Howell Jolly asplenia, sickle) lytes, BUN, Cr,
glucose, LFTs, TSH, B12, iron studies, CXR, ECG, urinalysis, sleep study,
colonoscopy for >50y
Normocytic:
Bilirubin/haptoglobin/LDH
(hemolysis), retic count, INR,
PTT, fibrinogen (DIC), DAT

Microcytic:
Ferritin, Fe, TIBC, FE
saturation (Fe deficiency,
anemia of chronic dz), anti-

Macrocytic:
RBC folate, B12,
TSH
(hypothyroid),

(hemolysis), SPEP/UPEP
(multiple myeloma), creatining
(renal failure), TSH,
AST/ALT/bilirubin/INR/PTT/albu
min (liver dz)

TTG/IgA (Celiac), Hb
electrophoresis (thalassemia),
serum Pb/erythrocyte
protophorphyrin (lead
poisoning)

AST/ALT/ALP
bilirubin,
INR/PTT
albumin (liver dz)

Heart Failure
Systolic HF (poor emptying): CAD/MI, HTN, dilatedCM, viral/toxins, aortic stenosis
Diastolic HF (poor filling): hypertrophic/restrictiveCM, mitral sten., pericardial dz
High Output Failure: anemia, Beriberi, thyroid, pregnancy, Paget's Dz, AV fistula
Arrythmia/Conduction: SVT, AF, AV block
NYHA HF
Classification
I No SOBw/ N activity
II @rest OK, SOB w/
N
III N activities limited
IV SOB at rest

CXR Findings for CHF


Alveolar Edema (LR+6)
Edema: Kerley B lines, periBronchial cuffing, vascular
markings (LR+12)
Cardiomegaly (LR+3.3, LR-0.33)
Dilated upper lobe vessel (pulm. veno. congestion LR+12)
Pleural effusion (LR+3.2)

Management
EKG, Echo, Angiogram, MIBI, urinalysis (proteinuria)
ACEi/ARB, beta blocker (hydralazine/nitrate if no ACEi, black, NYHA III/I)
Consider aldosterone antagonist
Symptom relief: nitrates, digoxin

75yo M experienced an episode of syncope/fall. Please take a history.


Orthostatic, vasovagal, carotid sinus, TIA/stroke, seizures, hypoglycemia,
hypovolemia, adrenal insuffienciency (postural), MI, VT/VF, afib, aortic dissection,
AS, HOCM, sick sinus/AV block, migraine, muscle weakness, vertigo, vision problem

1. HPI
Onset, duration, previous episode, memory of event, LOC
Pre standing up, emotional stimulus, lightheaded/diaphoresis, N/V,
urinating/defecating/coughing, head turning/putting on shirt/shaving,
reaching for object, exertion (AS/HOCM)
Headache, visual problem, dysarthria, dysphagia, ataxia, vertigo
CP/SOB, back pain, palpitations
Tonic-clonic, incontinence, tongue biting
Automatisms (lateralizations)
Post confusion, headache, imbalance
Bloody stool, diarrhea
Use of cane/walker
2. Meds/OTC/Allergies
BP meds/nitrates, antihistamine, diet pills
3. PMHx/Surgery/Hospitalization
DM (autonomic neuropathy), HTN, stroke, heart dz
Epilepsy, Parkinson's, anxiety

4. Social Hx
Smoking/ETOH/IVDU
Occupation, driving
5. Family Hx
Heart disase, neurological dz, osteoporosis
PEP Questions:

What blood tests would you do?CBC, lytes, Mg, Ca, BUN/Cr

What other investigations ? EKG, EEG, CT head, Echocardiogram

55yo M with headache. Take a focused history.


Stroke, SAH, subdural, migraine, cluter/tension, temporal arteritis, glaucoma, CO
poisoning, meningitis, brain abscess, brain tumor.

Offer turn down light (photophobia)


1. HPI
Onset, duration, trauma, timing (AM/PM), severity, quality
(throbbing, dull, sharp, tight), location (unilatera/bilateral),
progression, radiation, provoking/palliating (exertion, light, sound,
motion, NSAIDs)
Visual problems, aura, tinnitus, N/V
Dysarthria/dysphagia/ataxia/weakness, vertigo
Fever, chills, night sweats, neck stiffness, drowsiness
Scalp tenderness, jaw claudication
Low back pain (PMR = TA), sciatica
2. Meds/OTC/Allergies
Anticoagulants, nitrates, OCPs
Analgesics (rebound h/a)
3. PMHx/Surgery/Hospitalization
HTN, glaucoma, PMR
Head trauma
4. Social Hx
Smoking/ETOH/IVDU, occupation
5. Family Hx
Stroke, polycystic kidney dz, Marfan's, neurofibromatosis
55yo M complaining of dizziness/vertigo. Take a focused history.
Central vs peripheral vertigo: Central vertigo (often with neuro sx): tumor, stroke,
migraine, multiple sclerosis, Parkinson's; Peripheral vertigo: BPPV (sec-min),
labrinthitis (days), Meniere's dz (min-hrs; triad of tinnitus, hearing loss, vertigo).
Aortic stenosis, arrhythmia, orthostatic.

Asks if they need to lie down


1. HPI
Onset, duration, spinning, previous episode, timing (AM/PM),
provoking/palliating (standing up, looking up, closing eyes, staying
still)
Tinnitus, hearing loss, relationship to vertigo, N/V
Fever, chills, night sweats, wt loss, ear/resp infection

Headache, double vision, dysarthria, dysphagia, ataxia


CP, SOB, back pain, palpitations, syncope
Bleeding, bloody stools, bruising
2. Meds/OTC/Allergies
ASA, NSAIDs
Aminoglycosides, furosemide
Anticonvulsants
3. PMHx/Surgeries
DM, HTN, dLp, stroke, CAD
CHF, afib, aortic stenosis
Migraine, Parkinson's, MS
LMP, pregnancy
4. Social Hx
Smoking/ETOH/IVDU
5. Family Hx
Stroke, Meniere's dz
Mnemonics
BE SKIM (>3sx for 6mo): blank mind, easy fatigue, sleep, keyed up, irritable, muscle
tensions
MSIGECAPS (>5sx for 2wk): mood, sleep, interest, guilt, energy, concentration,
appetite, psychomotor, suicide; [post-partum, psychosis, stressors, seasonal,
bereavement (<2mo)]
GST PAID (>3sx for 7d): grandiosity, sleep, talkative, pleasure, activity, ideas,
distractibility
TRAUMA (>1mo): traumatic event, re-experience, avoidance, unable to function, 1
month, arousal increased.
OCD: unwanted obsessions, compulsions, insight
CAGE: cut down, annoyance, guilt, eye-opener

25yo M going crazy and can't take it anymore. Do a hx.


GAD, OCD, panic disorder, PTSD, social/specific phobia/agoraphobia, depression,
psychosis, somatoform d/o. R/O hyperthyroid, drug w/d, substance-induced,
pheochromocytoma

1. HPI
What he means, onset, duration, frequency, progression
BESKIM: Blank mind, energy, sleep, keyed up, irritable, muscle
tensions; panic attacks, phobia/social
OCD: Obsessions (recurring thoughts/worries), compulsions (hand
washing, repetitive patterns, checking doors/ovens/taps), special
order, concern about contamination, intrusivessness, insight into
thoughts,
SIGECAPS: Mood, SI/HI, self-harm, stressors, hallucinations
GSTPAID: grandiosity, sleep, talkative, pleasure, activity, ideas,
distractibility
TRAUMA: Trauma, flashbacks, nightmares

CP/SOB, diaphoresis, flushing, wt loss, palpitations, heat


intolerance
How is it affecting his life?
2. Meds/OTC/Allergies
Levothyroxine, SSRIs, ADD meds, decongestants
3. PMHx/Surgery/Hospitalization
Thyroid dz, CAD/CHF
Psychiatric hx
4. Social Hx
Smoking/ETOH/IVDU, MJ, cocaine, MDMA
Relationships, legal problems, work problems
Sexual hx (STDs)
5. Counselling
SSRI: takes 6 weeks. Trial 1 year. Escitalopram 10-20mg OD. A/E
nausea, diarrhea, libido. Assurance of effectiveness. Can do CBT.
PTSD: CBT, exposure, supportive.
OCD: SSRIs, clomipramine (TCA) w/ CBT, exposure and response
prevention
GAD: SSRIs/SNRIs, buspirone, buproprion w/ CBT
14 yo suicidal teen w/ bilateral bandages on wrists. Take a history.
Depression, anxiety, psychosis, substance abuse, eating disorder, personality
disorder, delirium, chronic pain, medication side effect.

Asks if they feel safe


1. HPI
Thoughts: onset, duration, frequency, stressors
Behaviors: Cutting/burning/scratching, letters/notes, wills, giving
away possessions
Actions: previous attempts, plans/rehearsal, firearms, why they are
thinking of it
SIGECAPS: Hopelessness, mood, loss of interest, sleep, appetite,
decrease concentration, wt loss/gain, energy
GSTPAID: grandiosity, sleep, talkative, pleasure, activity, ideas,
distractibility
TRAUMA: Trauma, flashbacks, nightmares
Anxiety, panic attacks, hallucinations/commands
Cold intolerance, constipation, wt gain
2. Meds/OTC/Allergies
Tylenol, TCAs, MAOIs
3. PMHx
Chronic dz/pain, cancer
Depression, psychosis, schizophrenia, personality d/o
4. Social Hx
Home: lives with parents, marital status (HEADSS)

Education: grade, like the teachers, feel safe


Activities: hobbies, extracurricular, clubs, sports
Drugs: smoking, MJ, IVDU
Safety: at home, any worry about abuse, suicide, self-harm
Sexuality: currently active, partners, condom use
Friends: anyone she can talk to
Legal Hx, violence, childhood abuse
5. FamHx
Suicide in family, psychiatric illness, substance abuse
25yo F comes in w/ rash. Treatment has not worked. Do a hx.
Acne, eczema/psoriasis, seborrheic keratosis, tinea, cellulitis, folliculitis, hives,
vitiligo, bullous pemphigoid, alopecia.
R/O actinic keratosis, basal cell carcinoma, melanoma, Steven-Johnson, TEN,
necrotizing fasciitis, pemphigus vulgaris.

1. HPI
Onset, duration, location, spread/change, seasonal changes,
provoking/palliating (heat, cold, sun, exercise)
Itch, pain, color, raised, scaling
Fever, chills, night sweats, wt loss
Joint/back pain, oral ulcers, hair/nail changes
Dysphagia, Raynaud's, SOB (CREST)
2. Meds/OTC/Allergies
Steroids, NSAIDs
Topicals
3. PMHx
Eczema/psoriasis, skin cancer
DM, HIV, autoimmune
4. Social Hx
Smoking/ETOH/IVDU
Travel hx, sun exposure, tanning beds
Sexual hx (STIs)
Pets/hobbies
5. Family Hx
Skin cancer hx
Eczema, psoriasis
PEP Questions:

Tinea corporis, how would you diagnose and treat? KOH scraping is
diagnostic. Fungal culture can also be taken. Topical azoles (ketoconazole,
clotrimazole, miconazole) for 2 weeks applied OD or BID.

Tinea: red with central clearing, maculopapular, oval-round, commonly on


trunk or face.

25yo M has a mole on his back. Take a history.


Melanoma, pigmented basal cell carcinoma, squamous cell carcinoma, seborrheic
keratoses, atypical moles, warts

1. HPI
Onset, change, asymmetry, border, color, diameter
Previous mole removal
Pain/itching/bleeding
>20 moles, freckling, sun burn/tanning
Fever, chills, night sweats, wt loss
CP/SOB
2. Meds/OTC/Allergies
NSAIDs
Psoriasis treatment
3. PMHx
Lesions (lentigo maligna), non-melanoma cancer, pancreatic ca
HIV, immunosuppression
Previous cancer treatment, previous psoriasis treatment
4. Social Hx
Smoking/ETOH/IVDU
Travel hx: sunny regions, sunscreen use.
5. Family Hx
Skin cancer (melanoma in 1st degree = 8x risk), mole syndromes
6. Counselling
Wear loose clothing, hat, sunglasses during summer, avoid noon
sun, high SPF screen, avoid tanning beds
Warn that during winter, especially skiing, still vulnerable
Suggest have someone look for moles and track them
Bad: irregular shape, bleeding, painful, not uniform color, ulcerating
PEP Questions:

What are the important characteristics about this lesion? ABCDE

What is the most important prognostic factor? Depth of lesion


Describe your management? Excision (possibly imiquimod or cryotherapy)
followed for 3 years q3months. If spread to Lns, chemo. If spread to bones,
radiotherapy.

3yo M is with a seizure witnessed by his mom. Take a history.


Febrile seizures (5Fs: fever, 5mo to 5 years, less than 15min, non-focal=generalized,
family history), epilepsy (>2 unprovoked), hypoglycemia, hypovolemia, meningitis,
breath-holding spells, migraine

1. HPI

Onset, duration, previous episode, LOC


Fever, cough, runny/stuffy nose
Nutrition/hydration, bleeding, head trauma, toxins
Pre standing up, emotional stimulus, N/V,
urinating/defecating/coughing, turning head, exertion (AS/HOCM)

Pre anxious, irritable, drowsy, stomach ache (epigastric)


CP/SOB, back pain, palpitations
Tonic-clonic, incontinence, tongue biting
Automatisms (lateralizations): eye movements, staring, hand
movements, lip smacking (make sure generalized, both sides)
Post confusion, headache, imbalance, weakness, lethargy,
paralysis (Todd's), behavioral changes (crying, aggressive)
2. Meds/OTC/Allergies
Abx
3. PMHx (Pediatric)
Previous seizures, unprovoked
Developmental hx, infections, brain injury
Prenatal: U/S, GDM, HTN, infection, IUGR, smoking/ETOH/drugs
GBS status, maternal UTI/fever, GA/delivery, APGARS, wt
NICU, metabolic screen, fever/shock/seizure/respiratory distress,
meconium
4. Social Hx
Parents occupation, marital status, safety at home, day care
5. Family Hx
Epilepsy, developmental delay
6. Counselling
Febrile seizure, no meds (except Tylenol)
If >5 min call 911, <30 min = no brain injury
Place baby on side, loosen clothing, clear area; DO NOT hold baby,
put things in mouth, put in bath tub
Follow up with GP
Rule of 5's: 5% kids have febrile seizures, 33% have recurrence.
Kids outgrow at 5 years. 2% at risk for epilepsy
Disordered electrical activity, epilepsy no cure (1/100), swimming,
bathing, triggers, driving, compliance with meds, alcohol lowering
seizure threshold
Atypical = >15min, recurrence, post-ictal confusion, weakness, focal
signs
PEP Questions
What can provoke seizures? Head trauma, infection, dehydration,
hypoglycemia, stroke, tumor, hyper/hyponatremia, hyperventilation
What investigations? CBC, LP, lytes, BUN, Cr/urea, CXR, BCX, U/A, brain
imaging, glucose, EEG if atypical. TSH. PRL (if 2x baseline after 10-20min)
Partial (one hemisphere): Simple = no altered LOC, aura; Complex: altered
LOC, automatism
Generalize (both hemispheres, LOC): altered LOC, tonic clonic, absence
Common conditions: ETOH/benzo withdrawal, cocaine, LSD, methanol,
ethylene glycol, TCAs, insulin, fevers in kids, meningitis, encephalitis,
hypoglycemia, hyponatremia, hypocalcemia, hyperthyroidism, mass, stroke,
trauma

5yo M has ear pain. Child has URTI. Mom want Abx. Counsel the mom.
AOM: acute otalgia, effusion (air fluid level, opacity, reduced bony landmarks,
ruptured TM, reduced mobility), inflammation (bulging with discoloration);OE: pain,
fullness/itching, red canal (w/ discharge, and TM can be red); OME: otalgia, ear
fullness, hearing loss, limited TM mobility, NOT red, air fluid levels.
Abrasion/laceration of auricle, cerumen impaction, foreign body, dental infection,
sinusitis, TMJ

1. HPI
Onset, duration, previous episodes, discharge (bloody, straw/watery)
Fever, coughing, sneezing, stuffy/runny nose
Sore throat, irritability, poor sleeping/feeding
Sick contacts, recent swimming, foreign body
Air travel, slap to ear, q-tip,
Neck stiffness, lethargy, HA, vomiting, seizure
2. Meds/OTC/Allergies
Pneumococcal vaccine
3. PMHx/Surgeries (Pediatric)
Cleft palate, Down syndrome, CF, immunodeficiency
Developmental hx, infections, brain injury
Prenatal: U/S, GDM, HTN, infection, IUGR, smoking/ETOH/drugs
GBS status, maternal UTI/fever, GA/delivery, APGARS, wt
4. Social Hx
Smoking in house, daycare, breast/bottle
5. Counseling
Explain that the majority of these, especially with URTI are viral.
If painful use Tylenol 15mg/kg PO q4h PRN (up to 5 doses)
AOM: waiting if >6mo, non-severe, previously healthy, and has f/u in
48hrs
If worsening at 48-72hrs use amoxicillin 75-90mg/kg/d, bid x510days
OE: topical abx + topical steroids (Ciprodex)
Come to a mutual understanding and agreement. Explain the
natural course of OM.
Red Flags: high fever, hearing changes, purulent discharge, not
going away.
Plz return if symptoms do not get better.
PEP Questions
What are the bacterial etiology of AOM? Strep pneumo, nontypeable
hemophilus, moraxella catarrhalis.
Otoscopic Findings
Otitis Media
Red bulging tympanic membrane, lose landmarks. ?mastoid pain
Otitis Externa
Swollen, moist, tender canal; Tug test
Serous Effusion
Amber color behind TM, maybe air bubbles
Retracted Eardrum Sharply outlined malleolar folds, protruding short process,
short/horizontal malleus handle

14yo F with short stature. Take a history.


Normal familial (normal bone age), constitutional delay; Abnormal hypothyroid,
malnutrition, Turner/down syndrome, GH deficiency, rickets, Cushing's, cancer,
newborn metabolic, cardiac dz, pulmonary dz, GI dz, renal dz. (Tall =
hyperthyroidism, GH excess, Klinefelter, XYY, Marfan's, homocysteinuria,
neurofibromatosis).

1.HPI
Growth %ile (<5cm/year, <3rd), when first started noticing
Headaches/visual changes, sexual maturity
Malnutrition: appetite, wt gain/loss, lethargy, pallor, bone fractures
Bleeding, nose bleed, joint swelling
Cold intolerance, dry skin, constipation
Infections/other: vomiting/diarrhea, oliguria/hematuria, SOB/cough
2. Meds/OTC/Allergies
Steroids, chemotherapy
3. PMHx/Surgeries
Infections, asthma, cancer, kidney dz
Gross motor: when did he walk, ride bike, climb stairs, throw ball
Fine motor: when did he grab objects with 2 fingers, draw a circle
Language: What age was first word, when started talking in
sentences, started obeying you, reading books
Social: does he play with other kids, does he share
Developmental regression, infections, brain injury
Prenatal: U/S, GDM, HTN, infection, IUGR, smoking/ETOH/drugs
GBS status, maternal UTI/fever, GA/delivery, APGARS, wt
4. Social Hx
Home: lives with parents, marital status (HEADSS)
Education: grade, like the teachers, feel safe
Activities: hobbies, extra-curricular, clubs, sports
Drugs: smoking, ETOH, MJ, IVDU
Safety: at home, any worry about abuse, suicide, self-harm
Sexuality: currently active, partners, condom use
5. Family Hx
Parental/sibling height, genetic syndromes
Developmental Milestones
Gross
Rolls B->F (3mo), Sit (6mo), crawl (10mo), walk (15mo), stairs
Motor
(18mo), tricycle (3yrs), bicycle (5yrs)
Fine MotorGrasps (3mo), transfer objects (6mo), pincer (10mo), scribble
(15mo), tower of 2 cubes (18mo), copies circle (3yrs), copies a
square (5yrs)
Language Coos (3mo), babbles (6mo), mama-dada (10mo), words (15mo),
2-word phrases (18mo), complete sentences (3yrs)
Social
Social smile (3mo), Stranger anxiety (6mo), peek-a-boo (10mo),
parallel play (15mo), No phase (18mo), group play (3yrs)

14yo F w/ mom. Concerned about her wt loss. Take a history


AN, bulimia, depression, social anxiety, GAD, panic disorder, OCD, substance use,
hyperthyroidism, DM, IBD, celiac, malabsorption, cancer

1. HPI
Onset, duration, max-min weight, typical meal, appetite, eats with
family.
Abdo pain, constipation/bloating
LMP: menarche, regular, length, amenorrhea
Binging/purging/exercising/fasting, body image, guilt, thought of
food, loss of control
Mood, appetite, interests, energy, concentration, sleep, SI
Heat intolerance, palpitations, diarrhea
Syncope/lightheadedness, seizures, hair loss
Perfectionist, good student
2. Meds/OTC/Allergies
Laxatives, appetite suppresants
Thyroid medications, SSRIs
3. PMHx/Surgeries
Depression, anxiety
Thyroid, DM
Seizures
4. Social Hx
Home: lives with parents, marital status (HEADSS)
Education: grade, like the teachers, feel safe
Activities: hobbies, extra-curricular, clubs, sports
Drugs: smoking, ETOH, MJ, IVDU
Safety: at home, any worry about abuse, suicide, self-harm
Sexuality: currently active, partners, condom use
PEP Questions

What would make you admit her? Syncope, lightheadedness, seizures,


palpitations, SI/self-harm, electrolyte abnormality, <80% body weight

What medication would you avoid for depression? Buproprion


AN Criteria

Description

Individual weighs <85%

Intense fear of gaining wt that is not alleviated by any wt loss

Distorted self-image of body wt/shape

Presence of amenorhea

Restricting type: restriction of calories/excessive exercise


Binge-geating/purging type: binge eating/purging

65yo F came. Son is worried about pt spending money. Take a history.


1. HPI
Reason for coming here, ask about money, where is she spending it
Living situation, friends/family, ADL/ IADLs, making meals
Mood, sleep, anxiety, concentration, suicide
Diarrhea/constipation, leak urine, falls, lightheadedness
Headache, CP/SOB, fatigue, fever, chills, night sweats
Memory, confusion, lost, hallucinations
MMSE: Orientation, repeat/recall 3 words, WORLD/backwards.
Clock drawing test. What she ate last night
2. Meds/OTC/Allergies
Cardiac meds, anticoagulants
Do you ever forget to take the drugs?
3. PMHx
Parkinson's, Alzheimer's/dementia
CAD, COPD, stroke
4. Social Hx
Smoking/ETOH/IVDU, hobbies, support
Living situation
5. Review: likely has Alzheimer's

Counselling (16 Stations)


35yo F PAP smear shows moderate dyskaryosis. Counsel her.
1. HPI
Last PAP, previous results
Vaginal discharge/bleeding, pain, itching, redness, abdominal pain
Menarche, regular, length of cycle/menses, irregular bleeding
Sexual activity, sexual debut, number of partners
presently/past/concurrent, men/women/both, oral/vaginal/anal
2. Meds/OTC/Allergies
Contraception/condoms
HPV vaccine
3. PMHx/Surgery/Hospitalization
STIs, HIV, PID, warts, cancer
Gynecological procedures, pregnancy, abortions
4. Social Hx
Smoking/ETOH/IVDU
5. Counselling
There are many types of HPV, some cause warts, other cause
cancer (such as cervical cancer).
HPV infection with high-risk types (HPV-16 and 18) cause over 99%
of cervical cancers. Most HPV infections go away without treatment
Can be transmitted without penetrative sex, due to contact in
anogenital region
HPV can stay latent, thus might not be found by PAP screen.
Gardasil (4types): protect against (HPV-16, HPV-18) that cause
cancer and (HPV-6 and HPV-11) that cause warts; Cervarix
(2types): HPV-16 and HPV-18 (70% of all cervical cancer)
Screening should begin at 21yo or 3yrs after first sex
PAP smear every 1 year. If 3 consecutive negative tests in 5 year
period, may be screened every 3 years.
If HSIL, AIS, cervical cancer, or immunosupression then continue
screening every 1 year.
ASC-US, LSIL: q6mo x12mo, if continue to be abormal referral for
colposcopy
ASC-H, HSIL, AGC, AIS: colposcopy
SCC, adenocarcinoma: consult
Prevention: reduce sexual partners, avoid concurrent partners,
condom usage, HPV vaccination, smoking cessation, PAP

16yo F came in asking for birth control. Counsel her.


C-OCP Absolute C/I: previous PE/CVA/DVT, CAD, HTN (>160/90), severe
hypercholesterolemia (also CI for progesterone only), thrombophilia/famhx, valvular
heart dz, smoker >35y (>15cigs/d), breast or endometrial cancer, abnormal vaginal
bleeding, abnormal liver function, pregnancy (also CI for progesterone only),
migraine with aura;

1. Sexual Hx
Previous use, current knowledge, personal preference
LMP, pregnancy
Vaginal discharge/bleeding, pain, itching, redness, abdo pain
Menarche, regular, length of cycle/menses, irregular bleeding,
Sexual activity, sexual debut, number of partners
presently/past/concurrent, men/women/both, oral/vaginal/anal,
current relationship, age of partner
2. PMHx
HTN, CAD, DVT/PE, liver dz/gallbladder
Cancer, clotting d/o, migraine with aura
Gynecological procedures, pregnancy, abortions
Past PAP results, last PAP, STIs
3. Meds
Antibiotics, anticonvulsants
4. Social Hx
Smoking/ETOH/IVDU
HEADSS
Childbearing goals? What you would do if you got pregnant
5. FamHx
Clotting d/o, cancer
6. Counselling
COCP and POP Pills 99% effective.
IUD 99% (lasts for 5 years). STI check before. Check for string
monthly.
C-OCP Pros: regulates periods, decreases risk of
endometrial/ovarian/GI cancer, helps with acne. CONS: vaginal
spotting first few cycles, bloating, weight changes.
POP CONS: compliance issue, must be taken same time each day.
IUD Pros: simple, long-lasting. CONS: uterine perforation.
COCP missed pill (if<12 hrs, take it). If >12hrs need to take for 7
days until covered again. Use condoms at this time.
POP missed pill (if<3hrs, take it). If >3 hrs take for 7 days.
Depo provera (every 3 months) shot option.
States that surgical sterilization is also an option, regret rate is 30%
F/u in 3 months
Most women get pregnant 3 months after stopping pills. May take up
to a year.

65yo F, personal and sensitive dyspareunia matter. Do a hx.


Superficial vaginal atrophy, vaginal spasm, vestibulodynia, candidiasis, chlamydia,
UTI, vaginal cancer, rectal cancer, endometriosis. Deep dyspareunia PID,
cervicitis, endometriosis, adenomyosis

Obtains consent and stresses confidentiality


I understand that this may be distressing, and I appreciate you
coming to talk about it
Sexual difficulties are quite common in otherwise healthy people

1. HPI
Onset (during/after coitus), duration (lifelong, other relationships),
quality (burning, aching, itchy), entry/deep, frequency of intercourse,
provoking/palliating factors (lubrication, position)
Vaginal spasm/dryness/discharge/bleeding, itching, redness, sores
Fever, chills, night sweats, abdominal pain/pelvic
LMP (menarche/regular/length), fatigue, hot flashes, mood, energy,
appetite, wt loss, sleep
Dysuria, constipation
2. Meds/OTC/Allergies
Viagra, HRT
SSRIs, beta-blockers
3. PMHx/Surgeries
DM, eczema
Anxiety
Fibroids/endometriosis, PID
PAP smear, infertility
Pregnancy, lacerations/episiotomies/trauma
4. Social Hx
Smoking/ETOH/IVDU
Sexual debut, # partners/concurrent, men/women/both,
oral/vaginal/anal, desire/arousal/orgasm
How they feel about their partner being involved, are they able to
discuss with them, are they happy
Because sexual violence is an enormous problem in our society
and can affect a person's health and well being, I now ask all my
patients about sexual health
Arguments, tensions, forced to have sex, sexually assaulted
5. Counselling
She has dyspareunia. Talk about hormone creams and reassure her.
Atrophic vaginitis due to lack of estrogen
First line treatment is estrogen replacement
Contraindications: breast/endometrial cancer, end-stage liver failure,
past history of estrogen-related throboembolization
Adverse events include: breast tenderness, vaginal bleeding, slight
increase in breast/endometrial cancer.
Routes of administation include oral, cream, transdermal.

Oral has benefit of preventing bone loss, and alleviation of hot


flushes. But may not alleviate vaginal dryness in 25% of patients
Transvaginal cream: very effective. Decreases UTI, lower hormone
amount. Increase in risk of endometrial cancer.
Transvaginal ring also possible.
Moisturizers and lubricants can be used in conjunction

14yo brought in by mom due to being withdrawn at home. Do a hx.


1. HPI
Onset, stressors: home, friends, school, what is the mom worried
about. Will have to speak to daughter alone in a bit.
SIGECAPS: mood, sleep, interests, guilt, energy, concentration,
appetite, wt gain/loss,
Anxiety, hallucinations
Home: lives with parents, marital status (HEADSS)
Education: grade, like the teachers, feel safe
Activities: hobbies, extra-curricular, clubs, sports
Asks mom to leave politely
Stresses confidentiality, unless you are at harm to yourself or others
Drugs: smoking/ETOH/IVDU, MJ
Safety: at home, abuse, suicide, self-harm
Sexuality: currently active, partners, condom use
2. PMHx
PAP smears, STI screening
3. Counselling
Elicits patient is worried about STD because of unprotected sex
Chlamydia, gonorrhea, HPV, HSV, HepB, syphilis, HIV, chancroid
Sexual debut, number of partners, concurrent, men/women/both,
oral/vaginal/anal, condom/OCP
Tattoos/IVDU, sex for money/drugs
Vaccine: HepA/B, HPV
Abdominal pain, discharge, itchiness, dysuria, lumps, red eyes
Sore throat, rashes, joint pain
LMP, cramping, pregnancy
Tells patient that betahcg will be done for pregnancy
Screen for syphilis, HIV, HBsAg, HepC Ab, endocervical swab (GC)
That if positive in Canada we HAVE to report chlamydia, gonorrhea,
syphilis, HIV. It is BC Law. Will be confidential.
Counsel patient on safe sex practices
Follow-up in 2 weeks

5yo M diagnosed w/ asthma/discharged w/ asthma meds. Counsel.


Encourages question, ensure understanding, ensure's mother's willingness and
ability to following

Airways very sensitive, causing narrowing and difficulty breathing.


Etiology: family history and environmental factors
Triggers: URTI, allergens, smoking, NSAIDS, emotional stress, cold
air, exercise.
Treatment is to manage not cure, most kids grow out of it, but can
be lifelong
Symptoms: night-time couging, difficulty breathing, wheezing.
Lifestyle: exercise, avoid allergens (pets, dander), avoid infections
Puffers: reliever meds (dilate airways, short-acting), controller meds
(inhaled steroids, help prevent attacks)
Steroids are not absorbed systemically, not to worry about S/E
Signs of attacks: difficulty breathing, pale/blue, fatigue, can't speak,
wheeze, change in mental status
During attacks give reliever puffers and come to ER if not improved
Ventolin for acute treatment: TID x1 week, then PRN
Ventolin's S/E: N/V, muscle aches, fine motor tremor, palpitations,
restlessness, throat irritation
Fluticasone for prevention: BID, everyday
Fluticasone's S/E: N/V, abdominal pain, diarrhea, headaches,
dizziness, nasal irritation/bleeding
Prednisone for decreasing inflammation: OD x 2 days
Prednisones S/E: N/V, mood changes, candidiasis, avascular
necrosis of the femoral head
Offers pamphlets and support groups
Books f/u to do PFTs
Instructs the mother about a follow-up visit

25yo F, sister dx w/ breast Ca. She is worried. Take a hx and counsel.


Cyst, fibroadenoma, fibrocystic changes, breast cancer, infection

1. HPI
Onset, number, size, association with menses, nipple discharge
(blood = benign intraductal papilloma), spontaneous/bilateral
discharge, skin changes/dimpling, pain
Fever, chills, night sweats, wt loss, anorexia, bone pain
SOB/CP, hemoptysis
LMP, menarche (<12 is early), menopause (>55 is late), first
pregnancy (>30y late), breast feeding (protective)
2. Meds/OTC/Allergies
OCP/HRT (>5yr @ risk)
3. PMHx/Surgeries

Previous mammograms/bx, radiation, breast cancer


Ovarian/endometrial/colon cancer
4. Social Hx
Smoking/ETOH/IVDU
High fat diet, exercise
5. Family Hx
Breast cancer, colon/ovarian/endometrial cancer
6. Counselling
I'll do a breast exam and teach you how to perform self breast
exams. [few days after cycle, offer chaperone if examining]
Discuss your sisters results with her
Mammography start screening at 50y q2yrs. For those with strong
family history start at 40y or 10 years prior to age of onset in first
degree relative
PEP Questions

Triple assessment: clinical exam, biopsy, imaging

Is the FNA diagnostic? NO. If non-bloody, and lump disappears = benign


cystic dz. If bloody, send for cytology.

What imaging would you consider? Diagnostic mammography, breast U/S


(identifying benign cysts) and guided core bx, MRI maybe for difficult lesions

What genetic tests would you consider? BRCA1/2 if family hx. Her-2 and
hormone receptor testing if mass is cancer.

Early stage: lumpectomy or mastectomy +/- RT then adjuvant therapy may


be offered based size, grade, lymph nodes, ER/PR/HER2

Breast Conserving Therapy: Lumpectomy + RT (preclude: multicentric dz,


large tumor size relative to breast, diffuse malignant-appearing calcifications
on imagin, prior hx of chest wall RT, pregnancy, persistent positive margins)

Mastectomy for those not candidates for BCT:

Medical tx: radiation, chemotherapy, or combined. Hormone and biologic for


hormone receptor +ve.

35yo F comes in asking about breast-feeding. Counsel her.


1. General
WHO recommends up to 6 months
Premature q2-3h, term q4h, q5h @ night until 4.5kg or 2-3 month
2. Benefits (ABCDEFGH)
ALLERGIC conditions reduced, BEST food for infants, CLOSE
relationship, DEVELOPMENT of IQ/jaw/mouth, ECONOMICAL,
FITNESS of mom (return to normal wt), GUARD against
breast/ovarian/uterine cancer, HEMORRHAGE reduction
Risk of HIV transfer
3. Contraindications
HIV/AIDS/active TB/herpes in breast
Heavy ETOH/illicit drugs
Chemo/rad compounds

Diazepam, metronidazole, lithium, tetracycline, bromocriptine,


chloramphenicol, ergots, gold, cyclophosphamide, antimetabolites
NOT C/I: mastitis, OCP, CMV, hepatitis
4. Complications
Sore/cracked nipples, engorgement, mastitis, poor wt gain, oral
candidiasis
Breast feeding jaundice, breast milk jaundice,
16yo M, Diabetes Type 1 is non-compliant with insulin regimen.
1. HEADSS
Home: lives with parents, marital status
Education: grade, like the teachers, feel safe
Activities: hobbies, extra-curricular, clubs, sports
Drugs: smoking, MJ, IVDU
Safety: at home, any worry about abuse, suicide, self-harm
Sexuality: currently active, partners, condom use
Elicits confession that they are a raging alcoholic
2. HPI
Mood, suicide
CAGE; Cutting down, Angry at criticism, Guilty, Eye-opener
Amount/week, types of alcohol, drinking more, drinking with friends
Effect on family/friends, trouble with police
W/D: tremor, mood changes, sweating, hallucinations
3. Counselling
Would they like to reduce amount?
Benefits: reduced risk of dz, improved mood, improvement in
function
Groups available -> Referal to alcohol team
Detoxification in community (Chlordiazepoxide over 1 week, reduces
symptoms) vs. in hospital
Reportable diseases in BC
Must be reported. Helps public
health determine trends and
monitors effectiveness of
interventions. It is mandated by BC
law.

Gonorrhea, syphilis, chlamydia, HIV,


Hep A/B/C, TB
HCP also needs to notify sexual
partners.

45M comes in with results. Please counsel.


SPIKES: setting, perception, invitation, knowledge, empathy, summarize

Postive TB sputum cx. Homeless. Postive HIV test. Married.


1. Setting
Puts on mask

Are you comfortable?

2. Perception (before you tell, ask)


Is it ok to share important news
If they want someone here
Their knowledge of the test
3. Invitation (obtaining patient's invitation)
Would you like me to give you all the information or sketch out the results
and spend more time discussing the treatment plan?
4. Knowledge (Unfortunately I've got some bad news to tell you)
Tested +ve for TB: infects lungs
causing cough and hemoptysis
HPI: cough, mucus, blood, fever
Patient asks how did I get it?:
homeless, alcohol dependent,
immunosuppressed are at risk
Do you know anyone else with TB?
There is a cure, 6 months of meds
Initial phase is 2 months (INH, RIF,
PZA, EMB), then AFB smears, then
4 months (INH and RIF)
A/E: liver dmg, vision rash, drug
fever
Long-term hospitalization for
treatment. Also Portland Hotel
Society.
Reportable, BC Law, confidential.

HIV test positive, not AIDS


HIV is a viral infection that attacks
the immune system and causes you
to have other infections
HPI: fever, lumps/bumps, thrush,
urethral discharge, pelvic pain, skin
rash, cough, mucus, blood, N/V,
diarrhea.
Patient asks how did I get it?:
sexual activity, sharing needles
Asks patient if they know how they
got it (IVDU, MSM, paying for sex)
No cure, treatments to keep virus
under control
Reportable, BC Law, confidential.
Any questions?

5. Empathy I can see how upsetting this is to you.


6. Summarize
TB is reportable.
How do you feel?
Summarize
Offers more information
Follow-up visit

HIV is reportable, inform


wife/sexual contacts, offer to help,
wife should be tested (public health)
Safe sex/abstinence to stop trans.
How do you feel?
Summarize
Offer more information
Follow-up visit

DNR Discussion (SPIKE)


1. Setting
I have something I would like to discuss with you
About your medical options, this is something I discuss with all my
patients.
Would you like someone present?

2. Perception (before you tell, ask)


What do you know about your disease
What do you expect out of your medical care, your goals for the
future.
What are things you would like to be able to do?
3. Invitation
Can we talk about CPR?
4. Knowledge
Do you know what CPR is?
CPR involved chest compressions to start your heart again
Most of the time it does not work
If you were to die suddenly would you like CPR?
What would be a reasonable quality of life that you would like to live
with
5. Empathy
What are your feelings around this topic?
We don't need to make our decision now, revisit this topic later
6. Summarize
If we place your status as DNR it does not mean withdraw care
In fact we would like to make you as comfortable as possible
I know this is a very difficult discussion, thank you for allowing me to
discuss it with you
We will talk again soon.
Geriatric Giant Screen
1. Drugs
What drugs/reason, S/E, forget
2. Delirium
Confusion, day/night, hallucination
3. Dementia
Forgetfulness, difficulty paying
bills, making meals, driving
4. Depression (MSIGECAPS)

5. Sleep
Sleep through, feel rested, snoring
6. Constipation
BMs, frequency, painful/hard
7. Urinary Retention
Incontinence, frequency, pain
8. Vision/Hearing
Visual problems, hearing speech

9. Falls
Falls, unsteady, lightheaded

10. Supports
Friends/family, fun/hobby

11. Functionality: ADLs: feeding, grooming, washroom; IADLs: cooking,


grocery, shopping, bills, driving
24yo F had labs show high chol. Counsel her.
Heart disease: arteries supply blood to the heart, sometimes they
can get clogged with plaque leading to a heart attack
TC, HDL, LDL, TG; high HDL is correlated with decreased CAD;
high LDL is correlated with increased CAD

Risk factors: male >45, female >55, DM/HTN, high cholesterol/low


HDL, obesity, smoking, FAMHx <60, inflammatory biomarkers
Screening: 40M q1-3 years, 50F/postmenopausal q1-3 years; >18
if(angina, DM, HTN, smoking, obesity, FMHx, CKD, SLE,
atherosclerosis)
Signs of hyperlipidemia: atheromata, xanthoma, tendinous
xanthoma, corneal arcus

1. HPI
CP/SOB(OE), orthopnea/PND
DM: Polyuria/polydipsia/polyphagia/wt loss
Cold intolerance, dry skin, constipation
RUQ pain, jaundice, frothy urine
2. Meds/OTC/Allergies
Steroids, beta-blockers
3. PMHx/Surgery/Hospitalization
DM/HTN, CKD, autoimmune
Pregnancy, GDM
4. Social Hx
Smoking/ETOH/IVDU
5. Family Hx
CAD <60
6. Counselling: She asks about statins and side effects.
Lifestyle changes, 3 month trial: decrease SAFA, increase fiber,
exercise, wt loss, smoking cessation, moderation of ETOH
Treat HTN and DM
Lab tests: HbA1c/glucose, Cr/urea, lytes, TSH, GGT
Statins: slow cholesterol formation (atorvastatin 10-80mg PO daily)
S/E include: <5% experience myalgia, also some get hepatoxicity
C/I: pregnancy
Monitor muscle enzymes and LFT (myopathy and hepatoxicity)
ALT, AST, CK at baseline, 6 weeks then q6mo
Niacin, bile acid-binding resins, fibric acid derivatives, ezetimibe
Lipids followed q3months
35 yo M asking about HTN and its treatment. Counsel her.
SBP (contracting heart), DBP (relaxing heart)
Normal is 120/80, HTN is >140/90 (DM 125/80, renal dz 130/80) of
>2 office visits
Can be white coat hypertesion, thus can also do ambulatory BP
Cause is likely artery hardening (but unknown), other secondary
causes are (Cushing's, pheo, aldosteronism, hyperthyroidism,
cocaine, aortic coarctation, renal artery stenosis)
Risk factors: age, obesity, sedentary lifestyle, stress, smoking,
ETOH, high salt, family Hx

Complications: stroke, blindness, heart attack, kidney failure


1. HPI
History of BP, previous treatment
Headache, weakness, visual problems, CP/SOB, claudication
Snoring, daytime somnolence, morning headaches (OSA)
Sweating, tremor, muscle weakness, tachycardia (pheo)
Heat intolerance, sweating palpitations, diarrhea (thyroid)
Thinning of skin (Cushing's), flank pain (kidney dz)
2. Meds/OTC/Allergies
NSAIDs, cocaine, sodium
3. PMHx/Surgery/Hospitalization
Sleep apnea
DM, high cholesterol
Afib, heart failure, MI
4. Social Hx
Smoking/ETOH/IVDU, cocaine
Work schedule
5. FamHx
6. Counselling:
Lifestyle changes x3months (exercises, salt, smoking/ETOH
cessation, stress reduction)
Lab work: HbA1c, creatinine/GFR, lipids, U/A, uACR, ECG
First line HCTZ or ACEi: diuretic that prevent body from retaining too
much salt
S/E: urination, dizziness, may increase blood sugars
F/U every 3 months
45yo F new to clinic with a history of Diabetes. Take a history.
1. HPI
Type I/II, when diagnosed, glucometer, levels in AM/PM, HbA1c
Hunger, dizziness, tingling, concentration
Polydipsia, polyuria, nocturia, polyphagia, wt loss
Blurred vision, impotence, constipation, bloating (gastroparesis),
orthostatic, feeling in feet
CP/SOB, claudication
2. Meds/OTC/Allergies
Insulin use
3. PMHx/Surgery/Hospitalization
Foot/eye care
Retinopathy, nephropathy, neuropathy
CAD/CVD, stroke, HTN, dLp
PCOS, gestational diabetes
Hospitalizations for DKA
4. Social Hx

Smoking/ETOH/IVDU, occupation, diet, exercise


5. FamHx
DM
6. Counselling
Diabetes is a disease of glucose dysregulation
What are risk factors? First degree relative, high risk population, hx
of IGT/GDM. Vascular dz, overweight, HTN, dLp, PCOS, acanthosis
nigricans.
Lifestyle trial for 3 months, then medications.
Firstline antihyperglycemic agents? Metformin, sulphonylurea,
alpha-glucosidase inhibitor.
Increases sensitvity of cells to insulin. OD/BID.
A/E include nausea, diarrhea, abdo pain, weight loss, lactic acidosis
C/I: kidney dz, low BMI
What are complications of diabetes? Neuropathy (impotence,
constipation, diarrhea, gastroparesis, orthostatic hypotension,
paresthetic feet), Retinopathy, Nephropathy, CVS, foot care,
hospitalizations (DKA, hyperosmolar nonketotic coma,
hypoglycemia).
PEP Questions
What are screening guidelines? q3yrs >40yrs no risk factors.
Diabetes: Symptoms + random glucose >11.1; FPG > 7.0; HbA1c > 6.5%

1yr M brought in by mother to discuss immunzations. Counsel her.


1. Education
Prevent serious illnesses. It is safe, majority of babies get it.
Also prevents spread of disease. E.g. smallpox eliminated
Does this make sense? Do you have any questions?
Inj: cry, irritable, low-grade fever, local reaction. Fluids+Tylenol
If large swelling, or around lips/mouth, rash, call doctor immediately.
Rare: 1/1000 high-grade fever, seizure, call 911. 1/500,000
anaphylaxis.
The disease is more srs than the complication, thus we vaccinate.
2. MMR and Autism
Paper published in 1990s in NEJM. Since been discredited.
The study was very small and the outcomes were biased.
Several large-scale epidemiological studies found no evidence.
3. Summarize
With your consent, you child will be vaccinated for these vaccines.
The reason is to benefit your baby and prevent any harm, as well as
for the interest of public health.
Vaccines are safe. There are minor side-effects such as slight fever
and irritability, use Tyelnol. If baby has severe reaction call doctor.

Does this make sense? Do you have any questions?

65yo M dx with Alzheimer's. Inform the patient and counsel him.


4 A's of Alzheimer's: amnesia, aphasia, apraxia, agnosia
SPIKES: setting, perception, invitation, knowledge, empathy, summarize

1. Setting
2. Perception (before you tell, ask)
Is it ok to share important news, if they want someone here, their
knowledge of the test
3. Invitation (obtaining patient's invitation)
Would you like me to give you all the information or sketch out the
results and spend more time discussing the treatment plan?
4. Knowledge: Unfortunately I've got some bad news to tell you
Positive for Alzheimer's, form of dementia: memory loss, confusion,
word finding difficulties, concentration, problems with paying bills,
getting lost etc..
Later changes include behavioral changes (sometimes aggressive,
sometimes passive), hallucinations, needing help with
eating/bathing/dressing, incontinence, driving will become
dangerous/cooking
Patient asks how did I get it?
Unknown mechanism, deposits in the brain, risks: age and family hx
No cure, medications improve memory, slow progression >10 years
Death is due to other illness (pneumonia, bladder/kidney infection,
complication of fall)
Meds are trialed for 8 weeks and monitored for side effects
For other symptoms such as depression we have SSRIs
Asks patient if he has any questions
Empathy
I can see how upsetting this is to you.
I guess anyone might have that same reaction.
Asks patient about social supports (family, friends)
Offers to help tell family
Summarize
Advises patient to think about getting financial affairs in order
Advises patient to think about drawing up an advanced directive
There are also support groups available for you and your family
F/U q3 months
35yo F wants to quit smoking. Counsel her.
1. HPI
Quantify cigarette use, previous attempts, health concerns, why they
want to quite (pre-contemplative, contemplative, preparation, action,
maintenance, relapse)
2. Knowledge

Pts knowledge of health risk; educates risks of smoking (stroke,


heart attack, sexual impotence, COPD, lung cancer, infections, other
cancers, costs)
Benefits of quitting (heart health, costs, better exercise tolerance,
better breathing)
3. Treatment
Counsels on quitting with friends or joining support groups;
Reassures the patient that it won't be easy; Advises that patient
takes on another activity (chew gum)
If failed, can try medications (Champix, wellbutrin, patch)
Stages of Change
Precontemplation
No intention to change behavior (4R's: reluctance,
rebellion, resignation, rationalizations).
Tx: provide information and raise doubt
Contemplation
Aware of problem, not made commitment (ambivalent)
Tx: explore the good and less good things
Preparation
Intending to change withing next month
Tx: help patient create their own plan and reinforce pros
of changing
Action
Making modifications to life
Tx: focus on success, reaffirm commitment
Maintenance
Prevent relapse
Tx: manage crises by reviewing progress that has been
made

SECTION II
The Strawberry-Jam Stations
(33 Stations)

Note: These are examples of the stations that usually occur in a couple
stations during the OSCE. They make you think oh S$!T

Combined Stations (4 Stations)


65yo M, transient loss/strain of vision L eye. Take a hx and px.
Closed/open angle glaucoma, vitreous hemorrhage, retinal artery/vein occlusion,
retinal detachment, cataracts, DM retinopathy, macular degeneration, optic neuritis,
migraine, CVA, temporal arteritis, trauma. HTN.

1. HPI
Onset, duration, bilateral, central/peripheral, total/partial, flashing
lights, floaters, lines
Pain, trauma, headache/temporal, dysarthria/dysphagia/ataxia
Weakness/paralysis/urinary incontinence (MS)
Red eyes, itchy, discharge
2. Meds/OTC/Allergies
Eye drops, heart medications
3. PMHx/Surgery/Hospitalization
DM, HTN, glaucoma,
Stroke, migraines, MS
4. Social Hx
Smoking/ETOH/IVDU
Occupation, driving
5. FamHx
Stroke, DM, migraines, eye dz
Physical Examination
Vital signs (commenting on afib)
1. Inspection
Symmetry, gaze, ptosis, trauma
2. Palpation
Carotid artery ausculation/palpation
Temporal artery (pain/beading), scalp tenderness, TMJ/open close
mouth (claudication pain)
Press on eye for pain (glaucoma)
3. Auscultation
Heart (S1,S2,S3,S4,murmurs)
4. Neuro
CNII: Visual acuity, color vision (decreased in optic neuropathies),
visual fields, pupils (PERL, RAPD),
Fundoscopy (papilledema, hemorrhage, vascular occlusion, cup-todisc ratio 0.5)
CNIII/IV/VI: EOM (nystagmus, dysconjugate gaze),
CNV, VII, VIII, IX/X, XI/XII
Finger-to-nose, repetitive movements, pronator drift
Tone, Motor/sensory/reflex, Babinski, clonus
PEP Questions:

What are the risk factors? Ocular hx, DM, HTN

What are some etiologies of vision loss? Glaucoma, retinal artery/vein


occlusion, optic neuritis, retinal detachment, temporal arteritis, retinopathy,
cataracts.
What imaging modality can be used? CT head scan if central process
suspected. Carotid doppler if suspicious for occlusion.
Picture shows hemorrhages, provide a differential? HTN retinopathy, vein
occlusion, papilledema, glaucoma, DM, trauma, macular degeneration,

45yo M comes in complaining of problems swallowing. Take a history.


Mechanical/motor/initiation. Stricture/Schatzki's Ring, esophageal carcinoma,
eosinophilic esophagitis vs. achalasia, esophageal spasm, scleroderma, GERD vs.
stroke, MS, myasthenia gravis, Parkinson's

1. HPI
Onset, duration, previous episode, progression (solids:mechanical,
liquids:motility), coughing/drooling/choking (initiation), food getting
stuck, pain, provoking/palliating factors (cold liquid:spasm)
Hoarseness, halitosis, heart burn/vomiting, regurgitation (Zenker's)
Fever, chills, night sweats, wt loss, appetite
SOB/cough, wheezing
Heat intolerance, sweating, anxiety, palpitations, diploplia
Joint pain/rash/skin thickening, Raynaud's (CREST)
Tremor, gait, balance, weakness/paralysis/incontinence (MS)
Fatigue with prolonged speaking, food in mouth after swallowing.
2. Meds/OTC/Allergies
Bisphosphanates, NSAIDs
3. PMHx/Surgery/Hospitalization
GERD (investigations), DM, cancer
Myasthenia gravis, scleroderma (CREST)
Stroke, MS, Parkinson's
4. Social Hx
Smoking/ETOH/IVDU, occupations
Suicide/caustic ingestion
5. FamHx
Cancer, MS, autoimmune
Physical Examination
1. Inspection
Distress, emaciation, oral cavity (dentition, abscess, pharynx), skin
(rashes, nodules/sclerosis), muscle (wasting/fasciculations)
2. HEENT
Palpate nodes, thyroid (nodules/goiters)
3. Chest
Tactile fremitus (aspiration = consolidation)
Auscultate lungs (vesicular, GAEB) [R/O aspiration pneumonia]
4. Neuro
Sit-to-stand (proximal), heel/toe walk, Romberg's
Easy fatigue: peek test = close eyes gently for 30 sec (MG)

Tone: hand tremor/cogwheeling/rigidity


CN V, IX/X, XI, XII, (no need for eye tests)
Pronator drift, finger to nose, repetitive movements

45 yo M w/ an incidental finding of hypercalcemia. Do a hx and px.


90% are malignant (lung, breast, myeloma, leukemia, lymphoma),
hyperparathyroidism (primary, lithium related, familial). Vitamin D related disorders
(toxicity, sarcoidosis), bone turnover (thiazides, hyperthyroidism, vitamin A
intoxication), renal failure (milk-alkali), thiazides, familial.

1. HPI (stones, bones, abdominal moans, psychic groans)


Onset, duration, lethargy, weakness, confusion
Polyuria, nocturia, polydipsia, flank pain/stones
Constipation, abdominal pain, nausea, anorexia
Fever, chills, night sweats, wt loss, appetite
Chronic cough
Syncope
Heat intolerance, palpitations, sweating, diarrhea
Amount of milk they drink.
2. Meds/OTC/Allergies
Vitamin D, vitamin A/tretinoin/acne,
Thiazide/diuretic, lithium
Calcium supplements
3. PMHx
Cancer, thyroid dz
4. Social Hx
Smoking/ETOH/IVDU
5. Family Hx
Parathyroid disorder, pancreatic cancer, thyroid cancer
MEN syndromes
Physical Examination
1. General
Confusion (AAOx3), band keratopathy (calcium precipitation), skin
turgor
Inspect thyroid, palpate for nodules
Palpate lymph nodes
2. CNS
Tone (low) and reflexes (sluggish)
3. CVS/RESP
JVP, heart auscultation,
Chest percussion and auscultation
4. Abdo Exam
Bruising (Grey-Turner's/Cullen's sign), bulging flanks,
Auscultates (BS, tinkling), aortic/renal bruits, venous hums,
Percuss liver, Castell's sign, shifting dullness, bladder distension,

Palpation: light/deep, rebound, liver edge, spleen, AAA


Testicular exam/pelvic exam

PEP Questions:
Lab tests: calcium, phosphorus, renal function, PTH, TSH, CBC, ACE
levels;
Imaging: CXR, bone scan

25yo M with a history of IVDU/traveled comes in with fevers. Do a px.


Infectious endocarditis (fever, track marks, heart murmur, heart failure, stroke, PE,
splenomegaly), cellulitis

1. HPI
Onset, duration, variations, fever, chills, night sweats, sweating,
myalgia, fatigue
Headache, neck stiffness, dysphagia/dysarthria/ataxia
New lumps/bumps
N/V, diarrhea
Cough, SOB/CP, stuffy/runny nose
Dysuria/discharge, back pain
Joint pain/back pain, skin rash, bloody stools
Abdominal pain/mass, jaundice
Heat intolerance, palpitations
Pregnancy
2. Meds/OTC/Allergies
Abx use, vaccinations
3. PMHx/Surgery/Hospitalization
TB, HIV, immunosuppression
RA, autoimmune dz
4. Social Hx
Smoking/ETOH/IVDU
Travel: where, when, activities, sick contacts, mosquitoes/ticks,
animal exposure (sheep, cattle, goats = Q fever: atypical
pneumonia, hepatitis)
Sexual hx, STD
Occupation (HCW, ID clinic)
Physical Examination
1. General
Distress, facial symmetry, jaundice
Janeway lesions, Osler's nodes, splinter hemorrhages, track marks
Palpates pulses in arm/feet
2. H/N
Lymph nodes, conjuctival hemorrhages
CNII-XII: EOM, pupils, fundoscopy =Roth Spot's
Finger-to-nose, repetitive movements, pronator drift
3. Cardiac/Resp

Auscultate carotids/palpate (bruits)


JVP
Thrills, apex beat, auscultate heart (murmurs)
Auscultates lungs
4. Abdo Exam
Angiomata, masses, bulging flanks, bruising
Auscultate for bruits and venous hum over liver
Palpate for hepatomegaly/splenomegaly, nodules, masses
PEP Questions

What are the immune complex signs? Osler nodes, roth spots
What are the vascular phenomena? Splinter hemorrhages, janeway lesions,
conjuctival hemorrhages

What are the organisms? Staph aureas (IVDU), streptococcus viridans and
staph epidermidis (prosthetic valve), enterococcus, HACEK and Candida.

What abx? PenG and Gentamicin (for streptococcus viridans and


enterococcus); Clox/Vanco and Gentamicin for staph aureus.

What 3 investigations? CBC, serial blood cultures, TEE

CXR, loculated opacity in RUL. What test would you order? AFB sputum,
sputum culture, thick and thin smear, Needle aspiration or biopsy >
transbronchial

Ddx: TB, granulomatous disease (sarcoid, Wegeners), malignancy, fungal


infection.

Physical Examination (5 Stations)


Pelvic Exam
Explain process and offers chaperone
1. Position
Feet in footrest, buttock at edge, drape mid abdomen to knee
2. Inspection of Externa
Labia majora/minora, urethral meatus, clitoris, vaginal introitus,
perianal region (inflammation, ulcers, warts, pustules, swelling,
discoloration, atrophy)
3. Internal Exam
Speculum: size, lubrication
Cervix: location, color, os, ulcers, lesions, bleeding, discharge
PAP smear
On removal assess for cystocele/prolapse
4. Bimanual Exam
Lubricate index/middle finger, palpate cervix, motion tenderness,
palpates uterus (bimanual) (size, shape, mobility), palpates ovaries
(size, shape, mobility).
5. Pelvirectal Exam
Tell patient and verifies consent
Middle finger into recturm (posterior vaginal wall = masses)
25yo M comes in with ear pain. Do a HEENT examination.
1. Head
Symmetry, size, shape, masses
Hair: quantity, distribution, loss, scalp redness, scaling, lumps, scars
Face: expression, hair, pigmentation, lesions
Palpates: scalp/skull for tenderness, temporal arteries (thickening,
tenderness, beading)
2. Eyes
Inspect: alignment, eyebrows, conjuctiva, sclera, cornea
Visual acuity (Snellen's), visual fields, EOM, PERLA
Fundoscopy (optic disc/cup, arterioles/veins, macula, fovea)
3. Ears
Auricle (size, position, symmetry), lumps, inflammation, discharge
Otoscopy (red/white, transparent/opaque, retracted/neutral/bulging,
cone of light, handle of malleus
4. Nose
Deformities, asymmetry, mucosa for swelling, exudate, ulcers,
polyps, septal deviation, inflammation, perforation
Palpates sinuses, performs transillumination
4. Throat

Lips: symmetry, color, crack, ulcers


Mucosa: color, ulcer, white patches
Gum: inflammation, ulceration, swelling
Teeth: missing, color
Tongue: movement, symmetry, ulcers
Uvula: ah, midline, symmetrical
Pillars: tonsils, color, symmetry, swelling, exudations, ulcerations

25yo M comes in to ER c/o severe left leg pain. IVDU. XR -ve. Do a px.
Septic arthritis vs osteomyelitis. Compartment syndrome: 6Ps - Pain out of
proportion, Paraesthesia, Pallor, Pulseless, Polar, Paralysis

ABCs
1. Inspection
Swelling, Erythema, Atrophy, Deformity (hip abduction, length of
leg), Skin (red, skin breaks, petechiae, surgical scars [CABG,
femoral-arterial bypass])
2. Palpation
Temperature, capillary refill, pulses, Baker's cyst
Effusion swipe test
Hip, femur, patella, tibial plateau, tibial tubercle, fibular head,
femoral head, tibia/femur.
3. ROM
If septic, will be limited due to swelling
4. Neuro
Tone, Motor (L2 hip flexion; L3 knee extension; L4 inversion of
foot; L5 extension of great toe; S1 eversion of foot)
Sensory (L2 lateral thigh; L3 middle knee; L4 middle ankle; L5
1st web space; S1 lateral foot) (light touch, pin-prick, vibration)
Reflexes (L4 patellar; S1 achilles, Babinski, clonus)
PEP Questions:

What test would you do? Bone scan.


Management of Fractures

Life before limb (ABCs), reduce fracture (prevent neurovascular injury then
XR and re-check status)

Irrigation/debridement/Abx for open fractures

Analgesics, immobilization/aids if required, tetanus prophylaxis

May require fixation, manipulation, arthroscopy

Red flags for f/u if: swelling, cyanosis, increased pain, decreased sensation

Complications: nonunion, malunion, joint stiffness, AVN, osteomyelitis

55yo F with claudicating pain/leg ulcer/tingling in feet. Do a px.


Spinal stenosis (radiating pain, worse w/ standing/walking, better w/ bending
forward/sitting). Arterial: vasculitis, popliteal (painful, rest pain, worse w/ walking,
better w/ rest, RF: smoking, HTN, dLp, CAD, MS, obesity). Venous (minimal pain,
pigmentation, marked edema, ulcer @ankles)

Comments on discomfort

Vital signs (BP in both arms, normal <10mmHg)


Lower Limbs- 1. Inspection
Feet, between toes, along soles
Shoes: room for toes, stones, sharps
Swelling, Erythema, Atrophy, Deformity (hammer toes, flat feet,
Charcot joint, gangrene, amputations), Skin (pink/pale, matted,
shiny/thin, surgical scars, hair growth, ulcers, necrobiosis lipoidica
diabeticorum [erythematous shins], diabetic dermopathy [thin,
atrophic, brown spots], ankle edema, calluses, fissures/cracks)
2. Palpation
Color, temperature, capillary refill, pulses (strength)
Perform ABI <0.9 = PAD
Pulses: popliteal, posterior tibial, dorsalis pedis
Squeeze calf for tenderness (ischemia)
3. Buerger's Test
Lift leg until pale, holding for 30s (if not pale test is normal). Now pt
sits up and hangs leg over bed (watch for 2-3 min, pallor followed by
hyperemia is positive)
4. Neuro Exam
Tone, Motor (L2 hip flexion; L3 knee extension; L4 inversion of
foot; L5 extension of great toe; S1 eversion of foot)
Monofilament testing (<6/10 = loss of sensation: 9 on plantar, 1 on
dorsal), vibration test (128hz), proprioception
Reflexes (L4 patellar; S1 achilles, Babinski, clonus)
5. Gait
Rombergs, heel/toe walk, tandem (broad based, foot drop, antalgic)
Upper limbs - 1. Inspection
Swelling, Erythema, Atrophy, Deformity (tendon xanthomata,
gangrene, amputations), Skin (pink/pale, matted, shiny/thin, scars)
Nicotine staining of fingers
Palpate temperature, capillary refill, radial/brachial/delay
2. H/N
Central cyanosis
Corneal arcus, xanthalasma (eyelid)
Carotid: auscultate, palpate (volume, amplitude, rhythm)
3. Chest/Abdo
Body habitus, scars, pulsations
Thrills, apex beat
Auscultate hearts
AAA, renal bruits
PEP Questions:

What are the signs of compartment syndrome? 6 P's Pallor, Pulselessness,


Pain, Paresthesia, Paralysis, Polar

What investigations? ABPI, duplex US, angiography, swabs, ECG, CXR,


HbA1c

Ulcers
Neuropathic
Appearance Red often w/ calluses
Pain
Tingling, numbness
Location
Below 1st MTP

Arterial
Red, punched out
Very painful
Shins and feet/toes

Venous
Dark ragged border
Little pain
Medial malleolus

60yo F with a 3 month hx of groin pain. Do a focused exam.


1. Inspection
Swelling, Erythema, Atrophy, Deformity (pelvic tilt, rotational
deformity), Skin (warmth, scars, lesions, wounds)
Hair on lower back, cafe-au-lait spots
Gait: heel/toe, tandem
2. Palpation
Spine/hip
3. ROM
Spine/hip
SLR, FABER
4. Neuro (radiculopathy, myelopathy, myopathy)
Tone, Motor (L2 hip flexion; L3 knee extension; L4 inversion of
foot; L5 extension of great toe; S1 eversion of foot)
Sensory (L2 lateral thigh; L3 middle knee; L4 middle ankle; L5
1st web space; S1 lateral foot) (light touch, pin-prick, vibration)
Reflexes (L4 patellar; S1 achilles, Babinski, clonus)
5. Abdo Exam
Symmetry, bulging flanks, masses, scars, bruising, angiomatas
Bowel sounds
Peritoneal signs, McBurney's, liver/spleen, DRE
CVA tenderness
Pelvic exam

History Taking (13 Stations)


35yo F comes in with hemoptysis. Take a focused history.
TB, PE, bronchitis, pneumonia, cancer, alveolor (Wegener's, Goodpastures), AVM,
bronchial arteries, pulmonary arteries, epistaxis, hematemesis, CHF,
thrombocytopenia, coagulopathy, trauma.

1. HPI
Onset, duration, previous episodes, color (clots, coffee grounds),
purulence, quantity, provoking/palliating factors
Fevers, chills, night sweats, wt loss, cough, runny/stuffy nose
Calf pain/swelling, SOB/CP
Nose bleeds, bruising, bloody stools, hematuria
Joint pain, rash, itchy eyes
N/V (blood), constipation/diarrhea, abdominal pain
2. Meds/OTC/Allergies
ACEi, anticoagulants
OCP
3. PMHx/Surgery/Hospitalization
TB, HIV
Lung dz/pneumonia
Cancer, cardiac/liver dz
Surgery in last 3 months
4.Social Hx
Smoking/ETOH/IVDU, cocaine
Recent travel, immobilization
TB/sick contacts
Occupation (HCW, prison, homeless), incarcerations
PEP Questions
CXR intial investigation, consider CT(PE)

What lab tests? CBC, lytes, LFTs, Cr, d-dimer, type/screen and crossmatch
What would you order for vasculitis? ANCA, anti-GBM, ESR, U/A
Bronchoscopy for normal CXR in males >50y with >40pyh w/ hemotypsis

Consult thoracic surgery

65yo M is a new patient with a history of COPD. Take a history.


FEV1/FVC <0.70. COPD, chronic bronchitis, emphysema, asthma

1. HPI
Onset, duration, night-time symptoms, provoking/palliating (activity,
smoke, cold air, NSAIDs)
Cough, sputum (blood, purulence), # COPDE/year [>4 red flag]
SOB: blocks/stairs walked, any SOB at rest
CP (rest/activity), orthopnea, PND, edema, wt gain
Fevers, chills, night sweats, wt loss

Facial fullness/pain, runny/stuffy nose, ear pain


Asthma, nasal polyps, ASA sensitivity (Samter's triad)
Eczema, rashes
Oral thrush
2. Meds/OTC/Allergies
Puffer use (>3x/week red flag), steroids, O2
Pneumococcal/influenza vaccine
3. PMHx/Surgery/Hospitalization
ER visits, ICU admissions
4. Social Hx
Smoking [>40pyh LR=12]/ETOH/IVDU
Missed work, pets, age of home, occupation
Social supports, activities
Plans for future, traveling, advanced directives
5. Family Hx
Atopy, CF
35yo M comes in with hematemesis. Take a focused history.
Peptic ulcer, esophagal varices, Mallory-Weiss tear, AVM, gastritis, esophagitis,
cancer, nose bleed, hemotypsis.

1. HPI
Onset, duration, episodes, frequency, content (coffee grounds, food,
bilious), strenuous vomiting, provoking/palliating factors
Epigastric pain (PQRSTPP), acid reflux, dysphagia
Jaundice, ascites, edema, dark urine/pale stools, itching
Nose bleed, bleeding gums, bruising, petechiae, bloody stools
Fever, chills, night sweats, wt loss,
Cough, CP/SOB,
Fatigue, tachycardia, light-headedess
2. Meds/OTC/Allergies
Anticoagulants, steroids, NSAIDs
3. PMHx
Liver dz, bleeding disorder
Surgery, hospitalization
4. Social Hx
Smoking/ETOH/IVDU
PEP Questions
Managements: IVx2, CBC, INR/PTT, group/x-match, lytes, LFTs

Monitor urine output


Endoscopy on unstable patients. Stable patients within 24 hours

Variceal bleeding (band ligation, sclerotherapy, tamponade).


Ulcer (endoclips w/ adrenaline, thermal coagulation, fibrin/thrombin)

Pantoprazole 80 mg IV bolus, followed by 8 mg/hour infusion 72 hrs postendoscopy. NPO for 8-12 hours post endoscopy.

65yo M comes in with a history of jaundice Take a history.


Pre-hepatic, hepatic, post-hepatic: spherocytosis, G6PD, sickle cell, HUS/TTP/DIC,
PBC, PSC, gallstones, pancreatic cancer, biliary cancer, infection (HepABC)

1. HPI
Onset, duration, previous episodes, itchiness, pale stool/dark urine,
ascites, confusion, hematemesis, bruising, melena
Abdominal pain (PQRSTPP), constipation/diarrhea
Fever, chills, night sweats, appetite, wt loss (ascending cholangitis)
Joint pain/skin bronzing (hemachromatosis), rash
SOB (alpha-1-antitrypsin)
2. Meds/OTC/Allergies
Statins, blood transfusions
Immunizations
3. PMHx/Surgery/Hospitalization
Liver dz, gallstones, RBC dz
Hemachromatosis, Wilson's
HIV, IBD
4. Social Hx
Smoking/ETOH/IVDU, tattoos
Sexual Hx, STDs
Travel history
5. Fam Hx
Liver dz, blood cell dz, hemachromatosis
65yo M comes in complaining of an abdominal mass. Take a history.
Cancer, abscess, hernia, cholecystitis, hepato/splenomegaly, hepatitis, pancreatitis,
AAA, ectopic, pregnancy, ovarian cyst, diverticular abscess, fibroids, bladder
distension, stool.

1. HPI
Onset, duration, size/progression, reducibility, location, pain
(changes with BM),
N/V, bloating, constipation/diarrhea, bloody stools/melena
Fever, chills, night sweats, wt loss
Dysphagia, early satiety, bone pain
Jaundice, fatigue, shoulder pain, dysuria/straining
LMP/pregnancy
SOB, cough, sore throat (splenomegaly)
2. Meds/OTC/Allergies
3. PMHx/Surgery/Hospitalization
Cancer, liver dz
4. Social Hx
Smoking/ETOH/IVDU
Travel hx
5. FamHx

Cancer, cysts, IBD


6. Counselling for CRC Screening
NO FamHx colonoscopy q10yrs @ 50yo OR FIT q1yr
If 1st degree relative <50yo then colonoscopy q5yrs @ 40yo OR
10yrs before earliest diagnosis
If 1st degree relative >50yo then FOBT q1yrs + colonoscopy q10yrs
starting @ 50yo
HNPCC: q1-2yrs colonoscopy @ 20yrs
FAP: q1yrs sigmoidoscopy @ 10yrs
45yo M comes in complaining of a neck lump. Take a history.
1 lump = cystic hygroma, congenital dermoid cyst, branchial cyst, carotid body
tumor, carotid artery aneurysm, salivary gland stone/tumor, thyroglossal cysts, single
thyroid nodule, goitre, pharyngeal pouch. If bilateral = chronic parotitis, mumps,
Sjogren/Mikulicz syndrome. If multiple = sebaceous cysts, lipomata, lymph nodes,
multinodular goitre

1. HPI
Onset, duration, location, number, growth, pain
Fever, chills, night sweats, wt loss
Dysphagia, odynophagia, hoarseness, halitosis
SOB/CP, cough, hemoptysis, wheezing, runny/stuffy nose
Palpitations, diploplia, heat intolerance,
Kidney stones, constipation, polyuria, fatigue
Ear pain
2. Meds/OTC/Allergies
Beta-blockers, lithium, thyroxine
3. PMHx/Surgery/Hospitalization
Pregnancy, thyroid dz, skin lesion (melanoma)
Radiation, cancer
4. Social Hx
Smoking/ETOH/IVDU
Travel: camping (lyme, fungal), developing countries (TB)
Sexual Hx, STD (HIV)
Cat scratches
5. FamHx
Thyroid dz/cancer, neurofibromatosis
65yo M comes in complaining of weakness
UMN vs LMN vs NMJ vs muscle (myopathy); Fatigue (weakness without an
anatomic or temporal pattern) vs Weakness (specific pattern with objective findings).
MS, myasthenia gravis, Eaton-Lambert, Guillain-Barre, ALS, spinal cord injury,
hypokalemia, hypercalcemia, hypo/hypernatremia, b12, hypothyroidism, Cushing's,
steroids, statins, ETOH, cocaine, interferon, infection (flu/EBV/CMV/lyme/HIV),
polymyositis, dermatomyositis, lupus, stroke/TIA.

1. HPI

Onset, duration, pain, fluctuation, distribution (diffuse, focal, one


side, proximal/distal), severity (rising from chair/brushing hair),
provoking/palliating (exercise [worsens with MG, improves with
Eaton-Lambert], heat [worsens with MS]), blocks you can walk
Numbness, tingling
Fever, chills, night sweats, wt loss, preceding illness (GBS)
Speech getting worse with prolonged speaking, drooling, chewing,
difficulty swallowing (MG: LR+4.5 LR-0.61)
Dysphagia, dysarthria, diploplia, CP/SOB (stroke/ALS)
Skin rash (SLE, dermatomyositis), joint pain
Cold intolerance, wt gain, dry skin, eye irritation
Mood, anxiety, suicide, sleep, hopelessness, stressors
2. Meds/OTC/Allergies
Steroids, statins, interferon
3. PMHx/Surgery/Hospitalization
Hypothyroidism, DM, adrenal, malignancy
CMV, HIV
Depression
4. Social Hx
Smoking/ETOH/IVDU,
Occupation (organophosphates in farming), lead exposure
5. Family Hx
MG, muscular dystrophy, autoimmune dz, collagen vascular dz, ALS
75yo F, refill on sleeping pills, do a hx and counsel her.
RLS, sleep apnea, pain/noise, bereavement/stress/depression/anxiety, dementia,
ETOH abuse, Parkinson's disease

1. HPI
Onset, trouble falling/staying/awakening from sleep, feeling
refreshed, leg twitching, snoring, nocturia
Medications, how much/how long, tried anything else
Mood, stressors, guilt, energy, concentration, suicide, confusion,
memory, incontinence, constipation, fall, pain, weakness
Elicits that patient's husband died 3 months ago and she started
having sleeping problems then.
Tells patient that she likely needs the medication for now
2. Meds/OTC/Allergies
3. PMHx
Lung dz, previous stroke
Myasthenia gravis
4. Social Hx
Smoking/ETOH/IVDU
Previous susbstance abuse
5. Counselling

Regular sleep schedule, avoid caffeine after lunch. ETOH/smoking


in evening, exercise before 7pm
DO NOT go to bed hungry/daytime naps/force sleep (get up after 20
min)
Use bed only for sleeping
Try some relaxation before bed
DO NOT use antihistamines
Medications
Benzodiazepines, non-benzo (zoplicone), trazadone (antidepressant), melatonin agonists
These medications may be habit forming
Can try for 4 weeks then reassess
Talk to doctor before you stop taking it [avoid withdrawal]
C/I in liver dz, avoid driving, avoid ETOH/sedatives/pain meds,
antihistamines
Side effects include: lightheaded, fatigue, weakness, change in
balance, blurred vision, confusion, dry mouth.
2wk old infant, fever of 38.8. Take a hx and counsel the mom.
Pneumonia, gastroenterits, UTI, meningitis, TORCH, autoimmune

1. HPI
Onset, duration, pattern, Tmax, thoughts of cause, lethargy, seizure
Coughing, SOB, wheezing, cyanosis, choking/poor feeding
N/V, diarrhea, foul smelling urine
Rashes, joint swelling
Jaundice, breastfeeding
Fluid intake/appetite, wet diapers, wt gain/loss,
Hearing/vision (TORCH)
2. Meds/OTC/Allergies
Immunizations
3. Pediatric Hx
Jaundice
Developmental hx, infection, prenatal care (U/S, GDM, HTN,
infection, bleeding), smoking/ETOH/drugs,
IUGR, GBS status/fever/maternal UTI, PROM
GA, delivery, birth wt, APGARS
NICU, metabolic screen, fever/shock/seizure/respiratory distress,
meconium
4. Past ObsGyn Hx
Previous, delivery, gender, wt, complications
5. FamHx
Congenital deformities, immunodeficiencies, sickle cell
PEP Questions:

What to do next? Find out no urine output for last 12 hours. Send to ER.

What three causes of sepsis in <1 month? Menigitis, UTI, pneumonia.


What are risk factors? PROM, GBS, chorioamnionitis, premature baby,
maternal UTI
What is a septic workup? CBC, blood cultures, U/A and Cx, LP, CXR.
Empiric antibiotic treatment with ampicillin and gentamycin should be
initiated.
Suspect a CNS or disseminated herpes simplex virus infection if the
neonate is lethargic, has mucocutaneous vesicles, seizures, elevated liver
transaminase enzymes, a negative CSF gram stain and/or active maternal
herpetic infection. In these cases, empiric acyclovir should be initiated.

9mo M accompanied by mother has been vomiting. Take a history.


Newborn: pyloric stenosis, esophageal atresia, duodenal atresia, hirschsprung, CF,
imperforate anus, necrotizing enterocolitis;
Infant: overfeeding, GERD, gastroenteritis, intussusception, malrotation, hernia,
toxin, meningitis, inborn error of metabolism.

1. HPI
Onset, duration, previous episodes, quantity, color/blood, projectility,
regurgitation/choking/coughing/arching
Abdominal pain, constipation/diarrhea (blood)
Fever, lethargy, toxic ingestion
Feeding, cow's milk (>24oz bad)
Urine output, wt gain/loss
2. Meds/OTC/Allergies
3. PMHx/Surgery/Hospitalization (Pediatric)
Developmental hx, infections
Prenatal: U/S, GDM, HTN, infection, IUGR, smoking/ETOH/drugs
GBS status, maternal UTI/fever, GA/delivery, APGARS, wt
NICU, metabolic screen, fever/shock/seizure/respiratory distress,
meconium
4. FMHx
Tract atresias, CF, hirschprungs, pyloric stenosis
PEP Question

AXR shows intussusception

What is the management of this condition? Air enema and/or surgery.

3yo M with respiratory distress. Take a focused history.


Bronchiolitis, asthma, pneumonia, epiglottitis vs croup, aspiration, foreign body,
allergic reaction, CF

1. HPI
Onset, duration, previous episodes, seasonal variation,
provokation/palliation factors
Fever, cough/barking, productive, cyanosis/wheezing, runny nose
Feeding, choking/arching, vomiting, rash
Poor weight gain/ear infections (CF)
Exercise intolerance/FTT (interstitial lung dz)
2. Meds/OTC/Allergies

Puffers, steroids, antibiotics


3. PMHx/Surgery/Hospitalization (Pediatric)
Asthma, pneumonia, CF
Developmental hx, infections, brain injury
Prenatal: U/S, GDM, HTN, infection, IUGR, smoking/ETOH/drugs
GBS status, maternal UTI/fever, GA/delivery, APGARS, wt
NICU, metabolic screen, fever/shock/seizure/respiratory distress,
meconium
4. Social Hx
Smoking, pets, molds, rugs, daycare
5. Family Hx
Asthma/atopy, CF
PEP Questions

What would you do for asthma: ventolin, ipratropium, inhaled and oral
prednisone, for severe asthma MgSO4

1wk old infant brought in has jaundice. Take a focused history.


Breast feeding jaundice, breast milk jaundice, prematurity, biliary atresia,
cephalohematoma, spherocytosis, G6PD, alpha-thalasemia, sepsis,
TORCH, congenital (Gilbert's, Dubin-Johnson, Crigler-Najar); hepatitis, CF.
1. HPI
Onset (<24hrs red flag), duration, progression,
breastfeeding/formula, difficulties with feeds, amount of feeding,
frequency,
Fever, lethargy, vomiting, coughing/wheezing (sepsis)
High-pitched cry/hypotonic (kernicterus), wet diapers/bowel
movements, dark urine/pale stools
2. Meds/OTC/Allergies
3. PMHx (Pediatric)
Developmental hx, infections, brain injury
Prenatal: U/S, GDM, HTN, infection, IUGR, smoking/ETOH/drugs
GBS status, maternal UTI/fever, GA/delivery, APGARS, wt
Cephalohematoma
NICU, metabolic screen (congenital hypothyroidism),
fever/shock/seizure/respiratory distress, meconium
4. Social Hx
Primary caregiver, financial situation
5. Family Hx
Spherocytosis, G6PD deficiency, biliary atresia, hypothyroidism
6. Counselling
Physiogical jaundice: diagnosis of exclusion, onset 2-3 days,
resolves at 7 days.
Breastfeeding jaundice: early onset, likely due to deprivation of milk
and dehydration. Exaggerated physiologic jaundice.

Breast milk jaundice: onset at 4-7 days, peaks 2nd to 3rd week,
unconjugated hyperbilirubinemia. Settle around weeks 4-5. R/O
dehydration as can make worse.

2yo M and he is pale. The Hb is 86. Do a focused history.


Microcytic (TAILS); Normocytic (cancer, autoimmune, IBD, infection, kidney dz,
aplastic anaemia); Macrocytic (b12/folate, hypothyroidism, hepatic dz, bone marrow
failure), reticulocytosis (blood loss, hemolysis, sickle cell).
Most common is iron deficiency anemia: too much cow's milk, malabsorption, cow's
milk protein intolerance.

6mo M comes in with growth failure. Do a focused history.


Decreased intake (breastfeeding, restrictive diet, neglect, cleft palate, eating
disorder, infection); increased demands/ineffective utlization (IBD, DM,
hyperthyroidism, adrenal insufficiency, cancer, CF, heart dz, renal dz, infection
HIV/TB/HBV/UTI), inadequate absorption (vomiting, pyloric stenosis, GERD,
Hirschsprung's, intussusception, hernia, biliary atresia, CF, cow's milk allergy, lactose
intolerance, IBD, celiac dz, short bowel syndrome, necrotizing entercolitis)
FTT: <3rd percentile, crosses 2 major percentiles. Kcal newborn: 100kcal/kg/day,
child 40-90kcal/day. Wt gain 0-3 mo is 25-30g/d, 3-6mo is 15-20g/day

1. HPI
Onset, breastfeeding (frequency, duration, both breasts, strong
latch, choking, cyanosis), iron source (breast/formula), cow's milk
(>24 oz), mother's diet, fatigue/pallor
Restrictive diet (vegetarian)
Fever, infections, wt loss/gain
Vomiting, diarrhea/constipation, abdo pain
Nose bleeds, bruising, bloody stools/vomit
Jaundice, dark urine/pale stools
Cough, SOB, lumps/bumps
Limping (rickets/leukemia), rashes (aplastic anemia)
2. Meds/OTC/Allergies
Septra, phenytoin/phenobarbital, steroids
3. PMHx/Surgery/Hospitalization (Pediatric)
Bleeding disorders, recurrent infections
Developmental hx, infections, brain injury
Prenatal: U/S, GDM, HTN, infection, IUGR, smoking/ETOH/drugs
GBS status, maternal UTI/fever, GA/delivery, APGARS, wt
NICU, metabolic screen, fever/shock/seizure/respiratory distress,
meconium
4. Social Hx
Parent's occupation, marital status
Neglect, daycare
Parent's height
5. FamHx
Sickle cell, thalassemia, spherocytosis

Consanguinity, developmental delay

PEP Questions (a)

CBC would show: hypochromic, microcytic.

Low ferritin, low serum iron, high TIBC, low transferrin saturation.

What other studies support the Dx? PTT, Hb electrophoresis, reticulocyte


count, sickle cell prep, B12/folate.

Appropriate management? Nutrition counselling, oral Fe

Counselling for family? Limit milk, eat more meat, avoid tea, increased Vit C

Child is at risk for? Leukemia, lead toxicity, developmental delay.


PEP Questions (b)

What would you look for on physical exam? Growth charts, dysmorphic
features, cataracts, cleft palate, thyroid, murmurs, poor AE, abdo
distension/mass, signs of abuse, tone, weakness, primitive reflexes.

What lab tests? CBC, lytes, Cr, BUN, ESR/CRP, U/A; LFTs, TSH, lipase,
albumin, glucose, ferritin, TIBC, immunoglobulins, sweat chloride, stool (fat,
reducing substances, C&S, O&P), ATTG, IgA. Bone age, karyotype.

What imaging? CXR, UGI scope, abdo U/S

8yo M comes in with the mother c/o of limping. Take a history.


Septic/reactive arthritis, juvenile rheumatoid arthritis (JRA/JIA), HSP, Kawasaki dz,
Legg-Calve-Perthes Dz, slipped capital femoral epiphysis (SCFE), Osgood Schlatter
Dz, developmental dysplasia of the hip, osteomyelitis, cellulitis. Leukemia, tumour,
trauma.

1. HPI
Onset, trauma, progression, provoking/palliating factors (NSAIDs)
Pain (PQRST, migration of pain, AM stiffness, night pain)
Fever, chills, night sweats, appetite, wt loss
Fatigue, pallor, nose bleeds, bloody stools, bruising [leukemia]
URTI/diarrhea, sore throat
Rash, back pain, incontinence
Sports
2. Meds/OTC/Allergies
Steroids
3. PMHx/Surgery/Hospitalization (Pediatric Hx)
JRA/JIA, sickle cell dz, leukemia
Bleeding disorders, recurrent infections
Developmental hx, infections, brain injury
Prenatal: U/S, GDM, HTN, infection, IUGR, smoking/ETOH/drugs
GBS status, maternal UTI/fever, GA/delivery, APGARS, wt
4. Social Hx
Marital status, physical/sexual abuse
5. Family Hx
Autoimmune joint dz
PEP Questions

If it is reactive, what tests would you do? Strep antigen, anti-DNAse B (for
strep), throat swab, U/A, urine C&S, FOBT, stool C&S

If you suspect osteomyelitis? Frog leg view of pelvis, bone scan.

What other tests would you consider? ANA, sickle cell, HLA-B27
(ankylosing spondylitis), complement factors, immunoglobulins, viral
serologies.

Counselling (11 Stations)


5yo M was diagnosed with Type 1 DM. Counsel the mom.
Type 1 due to lack of insulin, autoimmune, twin studies show 30%
concordance
Symptoms: polyuria/polydipsia/polyphagia, wt loss, dehydration,
blurred vision, N/V, abdo pain, lethargy -> DKA
Red Flags: N/V, abdo pain, fruity smell, coma, Kussmal respiration
Tx: very fast acting (Novorapid), long acting (Glargine), combination
2 regimens: biphasic (BID), QID (before meals +1 dose @ night)
Glucose checks: before meals, and before/after exercises
Option of Insulin Pump Therapy (constant basal rate, can also give
boluses); more control and can be empowering, covered up to 25y
HbA1c q3months
Long-term risks (>10yrs): neuropathy, retinopathy, renal failure,
CAD; Annual eye screening/foot checks
35yo F is pregnant, her sister had kid with Down's Syndrome. Counsel.
1. Counselling
Her chances are not increased by sister
Due to her age change is 1/350
Genetic counselling is available/financial support
2. HPI
Most common chromosomal condition (~1/800 births), screening
options (SIPS, IPS, FTS, NIPT[$800]), screening options not perfect
Diagnosis is by karyotyping
Increased materal age associated with DS
Facial features (upslanted eyes, protruding tongue, short neck),
palmar creases, hypotonic
Risk of cataracts, hearing loss, ear infections, cardiac anomolies,
OSA, GI tract anomolies, celiac dz, hypothyroidism, infertility, short
stature, obesity, leukemia
Developmentally delayed, behavioral problems, early Alzheimer's
3. Management
Multidisciplinary approach, most are born healthy but they need
close monitoring
Specialists include cardiologists and ophthalmologists, speech
therapist, dietitian, occupation and physiotherapists
Give child a normal of a life as possible
Support groups available
4. Prognosis

Despite the initial mortality rate being increased in the first year of
life, most children with Downs syndrome can expect to live until 5055 years.

Alice, daughter of your patient. Her mother has had Vfib, needed
resuscitation in ICU, and was comatose for a few days. Now AAOx3.
Alice wants a DNR.
Point out mother is alert, discuss with mother life-sustaining
treatments, point out mother's preference, it is mother's right to
decide.
Put herself in mother's shoes, she may welcome the discussion, can
be handled in sensitive way, daughter can be present during
discussion
Refuses to write DNR order, only with mother's consent, mother has
rights
Explain what the DNR order states.
What concerns she has? e.g. care-giver burden. Why does she
want her mother to be DNR? Also what does she think her mother
would want?
25yo F nurse stuck herself with a needle. Counsel her.
1. HPI
This must be a difficult to deal with, but I assure you it happens
quite often
Rinse with water (10min), disposed needle safely, reassure chances
of transmission are low
HIV (0.3%), HepB (30%), HepC (1.8%)
Depth of needle, blood on needle, patient IVDU/sexual hx
HepB vaccination, tested for HIV before
Consent patient and nurse for HepB/C, HIV testing
Talk to Worksafe
If nurse is vaccinated but no booster in last 6 months, give booster
If donor is at risk for HIV, start prophylaxis <72hours; Treatment is
28days; Taken twice a day
AE: N/V, diarrhea, myalgia, H/A, fatigue
Advise safe sex, and no blood donation
Nurse should be retested 1 month, 6 weeks, 3 month, 6 months
14 yo F came in for OCP. 2 days later mom comes in demanding chart.
Refuses to discuss, states information is confidential, it is the law
States mother and doctor visit also confidential
Did mom ask daughter about visit, is communication strained, offer
to counsel together
Daughter is competent, able to make own decisions

If confidentiality is broken, patients lose trust and will not disuss


important matters

35yo M diagnosed with pancreatic cancer, not operable. Wife comes in


to see you and asks you not to tell her husband the diagnosis. Counsel
her.
I can see that this news is very difficult and upsetting for you
Her knowledge of husbands condition, why she doesn't want the
husband to know
I understand the position you are in and your concerns are all valid,
but I have to respect his autonomy
What are you afraid of that might happen?
Patient will know something is wrong, will ask about diagnosis, will
lose trust, may want to put affairs in order, has right to know
Physician must tell the truth, if NOT asked will not provide more
information than they desire
If asked I will tell the patient
Refuses promise to not tell
This news is difficult for anyone to handle and we do have
resources in the community that are available to you such as
support groups.
2yo M is brought in by his mother fell against the table this AM and
cannot move his arm. Plain films shows a R clavicular fracture and an
old L rib fracture. Boy has extensive bruising. Discuss your concerns
with the mother.
She passive-aggressively argues with you about your decision to
admit the child and call the ministry.
1. RF for Child Abuse
Marital discord, age of parent when married, divorce, father/mother
drinks/drugs, parents abused as children, financial problems, son
has friends/handicaps
2. Physical Exam
Retinal hemorrhages
Skeletal fractures (spiral, bilateral, multiple, different stages of
healing, distal bone fractures, bucket handle fractures, mid shaft in
non-ambulatory infants, scapula, spinal fractures)
GU exam
Tx: admit, report, skeletal survery, PTT/INR, lytes, LFTs, CBC,
amylase, photos, CNS imaging if needed.
3. Management
Proper documentation
Proper imaging
Alert CAPS
SW consult

Spousal Abuse (SAFE)


Many patients are dealing with an abusive relationship, so I ask my
patients about domestic violence routinely
1. Safe
Do you feel safe in your relationship
Have they ever tried to kill you
Are there weapons in the house
2. Afraid
Have you ever been in a relationship where you were threatened,
hurt, abused
Physically abused/sexually abused
3. Friends
Are your friends and family aware that you have been hurt
Are you able to talk to them about this
Will they support you
4.Emergency Plan
Do you have a safe place to go
Do you have an escape route
Do you have an emergency kit
5. Closing
Thank you for discussing this with me
In a station with a Gen Surg resident. 2 abdo XR of man with colon ca.
Air fluid levels, dilated small bowel, thickened small bowel, mets in
the spine (no pedicles).
What is your diagnosis? SBO
What team would you consult? Gen Surg
CT scan done, not surgical candidate.
What drug class would you use in controlling his pain? Opioids.
What route? SC due to N/V
What dose? Patient already on PO, use card to convert to SC (half
PO dose).
How do you deal with N/V? 3 ways: NG tube, ondasetron, IV
metoclopramide with corticosteroids, bowel rest,
PEP: switch to fentanyl patch. Address 3 social problems (DNR,
Home Care, Palliative Care benefits).
Opiate Conversion
Codeine
Morphine
Oxycodone
Hydromorphone

100mg
10mg
5mg
2mg

T3 = 3mg of PO morphine
Morphine and
hydromorphone = PO:IV =
2:1

XR of partial SBO. Diagnose the XR and come up with a management.


Paralyitic ileus, pseudointestinal obstruction
Causes of LBO: colonic cancer (50-60%), acute diverticultisis (40%),
volvulus, fecal impaction.
1. HPI
Abdo pain (PQRSTPP)
Abdo distension, discomfort, N/V, no BM, no flatus
Food (seeds/popcorn)
Fever, chills, night sweats
Bloody stools
2. Meds
Opiates, anticholinergics
3. PMHx/Surgery/Hospitalizations
Abdo surgery, IBD, colon ca, hernia, food (seeds/popcorn)
4. Physical
Vitals, abdo distension, tenderness, peritoneal signs, high pitched
bowel sounds
5. Labs
hypovolemic hypochloremic hypokalemic metabolic alkalosis
XR findings
>=3 distended loops of SB, >=3 air fluid levels on upright film, >3mm
bowel wall thickening
String of pearl sign (mechanical obstruction)
R/O perforation (free air over liver, or in lateral decubitus)
Large bowel has haustra and is larger in size
Management
NPO, IV hydration, analgesia, NG tube, foley, CT abdo
Surgery after 12-24hr of NO improvement
Dimenhydrinate 25mg PO q4h PRN
Metoclopramide 12.5mg PO q4h PRN
Ondansetron 4mg PO q12h PRN
You are a resident and you are taking phone call from a nurse. A
patient with Alpha Thalassemia w/ A+ receiving a blood tranfusion. He
is having dyspnea, chest pain, and in distress.
Stop transfusion
Get an EKG, trops, CXR
Take a history.
Fill out questions about etiology (AHTR, TRALI, TACO), and
pathophysiology and cause of AHTR.
Descriptions
AHTR, acute hemolytic transfusion reaction (1:25,000), destruction
of donor RBC; fever, flank pain, red/brown urine, shock; stop
transfusion, infuse NS, alert blood bank; BW: DAT, free hemoglobin,
antibody screen, urine for hemoglobin

TRALI, transfusion-related acute lung injury (0.5-2/1000), activation


of WBC leading to capillary leak and pulmonary damage; SOB,
fever, hypotension, non-cardiogenic pulmonary edema (no JVP, lung
crackles); CXR, BNP; stop transfusion, oxygen, mechanical
ventilation, IV fluids, vasopressors, NO diuretics
TACO, transfusion associated circulatory overload (1:100), increase
in CVP, increase in pulmonary blood volume, secondary CHF,
elevated JVP, crackles, hypertension, cough, orthopnea, pedal
edema; CXR, BNP, stop transfusion, O2 and diuretics
TRALI

TACO

Temp

Fever maybe

Unchanged

BP

Hypotension

Hypertension

Resp

SOB

SOB

Neck Veins

Unchanged

Maybe distended

Auscultation

Rales

Rales +/- S3

CXR

Diffuse, BL infiltrates

Diffuse, BL infitrates

Pulmonary edema

Exudate

Transudate

Diuretic

NO

Responds

WBC

Transient leukopenia

Unchanged

BNP

<200

>1200

CXR with left pneumothorax.


Describe: increase lucency in lung field, pleural line viscera away
from CW, atelectasis, tracheal shift
Ddx: aortic dissection, MI, PE, esophageal rupture, fractured ribs,
tension pneumothorax
Conservative treatment: if pt not in distress, <25%, no underlying
lung pathology, hypoxia, dyspnea, pain
Invasive: simple aspiration if >25%, CT drainage if fail, +/pleurodesis

Section III
The Appendix
Drug Counselling
SSRI

Antidepressant, affects NT called serotonin, thought to play a part in


depression

Escitalopram 10-20mg PO OD. Begins working at 6 weeks. Take for at


least 1 year.

A/E: diarrhea, nausea, appetite, weight change, headache, w/d

C/I: suicide risk


Bisphosphanates

Prevents bone loss (supplement to exercise, no smoking, Ca/D)

Full glass of water, 30 min before food, upright for 30 min

Risk of osteonecrosis of jaw, dental checkups

A/E: headache, heartburn/bloating, diarrhea/constipation, abdo pain

C/I: pregnancy, dysphagia, renal disease


Warfarin

Thins blood by blocking vit K

OD. Start with 5mg for 4 days then INR on days 5 and 8

3 months for DVT, 6 months for PE, AF = lifelong

2-3 days before starts working, therefore bridging with LMWH

A/E: bleeding (dark stools, bruises, longer healing), diarrhea, rash, nausea,
interacts with other drugs

C/I: pregnancy, hemorrhagic stroke, bleeding, high falls risk


Levothyroxine

Synthetic thyroid hormone

OD, review in 2-3 weeks. TSH q2-3 months. Then annualy

A/E: if too much diarrhea, headache, heat intolerance, palpitations


Iron Tablets

Replace store of iron, helps make RBCs

Works best on empty stomach with orange juice. May irritate though.

3-4 weeks to bring up Hb then 3 more months to fill up stores.

A/E: nausea, diarrhea, dark stools, bad taste, constipation


Lithium

Stabilizes mood

Regular review by psychiatrist

Blood work: glucose, TSH, betahcg, CBC/lytes/renal, ECG

Check lithium levels after 4 days, then weekly until stable for 4 weeks, then
every 3 months.

Check TSH, CBC/lytes, Ca every 6 months

A/E: nausea, abdo pain, metallic taste, tremor, thirst, polyuria, wt gain,
edema.

Toxicity: dysarthria, ataxia, muscle twitching tremor, drowsiness, renal

toxicity, nephrogenic diabetes, hypothyroidism


C/I: 1st trimester pregnancy, breastfeeding, renal impairment,
hypothyroidism
Methotrexate

Reduces inflammation suppresses immune system

Once weekly with folic acid at a separate time

Takes 4-6 months to work

BW: glucose, LFTs, CBC/lytes

A/E: alopecia, headaches, N/V, abdo pain, myelosuppression, liver toxicity,


pulmonary toxicity

C/I: Pregnancy (even males), liver damage, breastfeeding


Atypical Anti-psychotics

Blocking receptors involved in schizophrenia

Daily or depot injections q2-4 weeks

BW: Lfts

A/E: tardive diskinesia, tremor, constipation, dry mouth, dizziness,


drowsiness, hypotension; NMS, agranulocytosis

C/I: liver failure

Other C/I: DM, Parkinson's, kidney problem, pregnancy


Levodopa

Replace dopamine, given with carbidopa

3-4 times a day with food

Effect weakens at 5 years

A/E: psychosis, N/V, hypotension (but there are drugs to help)

C/I: glaucoma

Levels of Evidence from Canadian Task Force on Preventive Health Care


I
RCT
II-1 Controlled trial(s) without randomization
II-2 Cohort or case-control studies
II-3 Multiple time series designs with or without the intervention, also uncontrolled
trials with dramatic results
III
Opinions, clinical experience, case studies, report of expert committees
Grades of Recommendations
A
Good evidence benefit (outweighs risk)
B
Fair evidence benefit (outweighs risk)
C
Conflicting evidence (clinician need not offer)
D
Fair evidence against (should not offer)
I
Insufficient evidence (help patient understand the uncertainty)

You might also like